NRSG 225 Practice Questions for Final (Potter and Perry 8th Edition) – Flashcards

Unlock all answers in this set

Unlock answers
question
12. Client assessment provides the nurse with necessary information for the development of an effective plan of care. When determining the influence of an internal variable on the client's health status, the nurse will specifically look for: 1. Anxiety level present 2. Family remedies used 3. Location and type of occupation 4. Available health insurance coverage
answer
Anxiety level present
question
17. The nurse knows that the greatest internal factor to consider when educating an adult client concerning health promotion activities is the client's: 1. Emotional wellness 2. Developmental stage 3. Professed spirituality 4. Intellectual background
answer
Intellectual background
question
10. A client with chronic respiratory problems tells the nurse, "I haven't felt this good in a long time." The nurse realizes that the statement most reflects the client's: 1. Willingness to share his feelings 2. Acceptance of his physical limitations 3. Personal definition of his individual health 4. Acknowledgment of his chronic health problems
answer
Personal definition of his individual health
question
20. Which of the following nurse-family interactions is most reflective of caring for the family? 1. Offering to arrange for a sleep chair for the family's use 2. Notifying the family that the client has returned from surgery 3. Telling the family when the client's surgeon will be on the unit 4. Always being available to spend time answering the family's questions
answer
Telling the family when the client's surgeon will be on the unit
question
12. Which of the following statements reflects the best understanding of cultural caring provided by professional nurses? 1. "Nurses must be open to learning the culture of our clients." 2. "Nurses need to attend to clients in a culturally sensitive manner." 3. "I care for my clients in ways that respect their culture and beliefs." 4. "Culture caring means allowing the client the freedom to be himself."
answer
"I care for my clients in ways that respect their culture and beliefs."
question
21. With which of the following interventions does the nurse best reflect caring by maintaining belief in a client? 1. Offering a client with cancer pain medication before a family visit 2. Explaining to a client what to expect during a bone marrow aspiration 3. Arranging for a burn client to talk with others who survived similar burns 4. Explaining to a client that he may select from a variety of entrees for dinner
answer
Arranging for a burn client to talk with others who survived similar burns
question
1. A nurse who wants to apply a theory that relates to moral development should read more from the work of: 1. Gould 2. Freud 3. Erikson 4. Kohlberg
answer
Kohlberg
question
2. The nurse using Erikson's theory to assess a 20-year-old client's developmental status expects to find which of the following behaviors? 1. Coping with physical and social losses 2. Enjoys participating in the community 3. Applying self to learning skills 4. Overcoming a sense of guilt or frustration
answer
Enjoys participating in the community
question
3.The nurse recognizes that Freud's theory approaches development by looking at: 1. Moral reasoning. 2. Logical maturity 3. Psychosexual aspects 4. Cognitive development
answer
Psychosexual aspects
question
4. According to Piaget, a preschool child (3 to 5 years old) who comes to the clinic is expected by the nurse to exhibit which of the following behaviors? 1. Far-reaching problem-solving 2. Exploration of the environment 3. Cooperation and sharing with others 4. Thinking with the use of symbols and images
answer
Thinking with the use of symbols and images
question
7. The nurse recognizes that which one of the following statements about growth and development is correct? 1. Development ends with adolescence. 2. Growth refers to qualitative events. 3. Developmental tasks are age-related achievements. 4. Cognitive theories focus on emotional development.
answer
Developmental tasks are age-related achievements.
question
9. According to Piaget, the infant is in the first period of development, which is characterized by: 1. Concrete operations 2. Preoperational thought 3. Sensorimotor intelligence 4. Identity versus role confusion
answer
Sensorimotor intelligence
question
4. A 49-year-old client is experiencing problems with depression. She has come to the clinic showing signs of malnutrition and fatigue. Which of the following is the best initial statement for the nurse to make in the assessment phase? 1. "How much weight have you lost over the past month?" 2. "Have you recently been experiencing menopausal symptoms?" 3. "Depression is something to expect at your age, and with assistance you will get better." 4. "Your depression is somewhat uncommon. Can you tell me what has happened recently to cause it?"
answer
"Have you recently been experiencing menopausal symptoms?"
question
8. A nurse is working in the health office at a local college where most of the students are young adults. Being aware of the major concerns for this age-group, the nurse includes assessment of these clients': 1. Current marital history status 2. Lifestyle and leisure activities 3. Experience with chronic disease 4. History of childhood accidents
answer
Lifestyle and leisure activities
question
12. The nurse, preparing to discharge an 81-year-old client from the hospital, recognizes that the majority of older adults: 1. Require institutional care 2. Have no social or family support 3. Are unable to afford any medical treatment 4. Are capable of taking charge of their own lives
answer
Are capable of taking charge of their own lives
question
13. To assist older adults to meet their needs for sexuality, the nurse should recognize that the greatest impact on the sexuality of older adults is: 1. Therapeutic medications may alter sexual function 2. Sexual interest declines and then fades completely with age 3. Physiological changes do not adversely influence sexual activity 4. Prevention of sexually transmitted diseases is no longer an issue
answer
Therapeutic medications may alter sexual function
question
16. In the assessment of older-adult clients, it is often difficult to discriminate between delirium and dementia. Delirium is characterized by: 1. A slow progression 2. Lasting months to years 3. A normal state of alertness 4. Occurrences at twilight or darkness
answer
Occurrences at twilight or darkness
question
17. Which of the following nursing questions is best directed towards the assessment of a normal finding regarding physiological changes in an older-adult client? 1. "Any difficulty driving at night?" 2. "Are you experiencing any loss of libido?" 3. "Do you see yourself as becoming forgetful" 4. "Have you had your cholesterol tested lately?"
answer
"Any difficulty driving at night?"
question
20. The nurse is planning client education for an older adult being prepared for discharge home after hospitalization for a cardiac problem. Which nursing action addresses the most commonly determined need for this age-group? 1. Suggest that he purchase an emergency in-home alert system. 2. Arrange for the client to receive meals delivered to his home daily. 3. Encourage the client to use a compartmentalized pill storage container for his daily medications. 4. Provide a written document describing the medications the client is currently prescribed.
answer
Encourage the client to use a compartmentalized pill storage container for his daily medications.
question
22. Which of the following client statements regarding self-medication administration by an older-adult client requires follow-up teaching by the nurse? 1. "I take all the pills ordered once a day at bedtime, so I'm less likely to forget them." 2. "I have one pill that needs cut in half. I am going to ask the pharmacist to do that for me." 3. "The pharmacist said to keep my pills away from the sunlight, so I put them inside the kitchen cabinet." 4. "My daughter comes over each morning and puts my pills into a container that sorts them by the time they are due."
answer
"I take all the pills ordered once a day at bedtime, so I'm less likely to forget them."
question
23. Which of the following statements made by an older-adult client poses the greatest concern for the nurse conducting an assessment regarding the client's adjustment to the aging process? 1. "I use to enjoy dancing and jogging so much, but now I have arthritis in my knees so that it's hard to even walk." 2. "I've given my grandchildren money for college so they can live a better life than I had." 3. "Growing old certainly presents all sorts of challenges. I wish I knew then what I know now." 4. "As I age I've found its harder to do the things I love doing, but I guess it will all be over soon enough."
answer
"As I age I've found its harder to do the things I love doing, but I guess it will all be over soon enough."
question
26. Which statement made by an older adult would reflect the best understanding of the nutritional guidelines for this age-group? 1. "I can prepare grilled chicken at least 10 different, delicious ways." 2. "When I entertain, I serve healthy foods like veggies and low-fat dip." 3. "I know I need to eat nutritiously, and I have certainly been doing better." 4. "I take seriously the suggestions my health team gives me on healthy eating."
answer
"When I entertain, I serve healthy foods like veggies and low-fat dip."
question
28. Of the following options, which is the greatest barrier to providing quality health care to the older-adult client? 1. Poor client compliance resulting from generalized diminished capacity 2. Inadequate health insurance coverage for the group as a whole 3. Insufficient research to provide a basis for effective geriatric health care 4. Preconceived assumptions regarding the lifestyles and attitudes of this group
answer
Preconceived assumptions regarding the lifestyles and attitudes of this group
question
29. The nurse is preparing an educational program for members of the local senior center. Which of the following topics would present the greatest learning challenge for this developmental group? 1. Exercising arthritic joints 2. Tips for living with GERD 3. Importance of the human touch 4. Principles of heart-healthy eating
answer
Importance of the human touch
question
31. Of the following client statements made by an older adult client which best reflects an understanding the educational materials on nutrition presented by the nurse? 1. "I'll keep this literature and read it again later." 2. "I love rye bread. It's good to know its high in fiber." 3. "Nutrition and cooking has always been passions of mine." 4. "Now I can see the connection between food and my health."
answer
"I love rye bread. It's good to know its high in fiber."
question
33. Which of the following statements made by a nurse best reflects an understanding of the negative impact of ageism regarding client care? 1. "If I don't value the older client, I will never be able to provide the care they are entitled too." 2. "Everyone, regardless of age or position, always deserves effective, appropriate nursing care." 3. "As a society we lose so much valuable wisdom and knowledge when we devalue our older members." 4. "If older clients do not feel valued, they are less likely to seek the health care they need and deserve."
answer
"If I don't value the older client, I will never be able to provide the care they are entitled too."
question
34. Which of the following statements made by a nurse best reflects an understanding of the adaptation required of nursing to assure quality nursing care for the older adult client? 1. "Remember to ask the client when she prefers to have her bath." 2. "I hope that I am that alert and interested in life when I'm her age." 3. "My client is in her 90s, so I don't expect her to respond to the therapy like a 50-year-old does." 4. "I just finished reading a great article on caring for the client newly diagnosed with Alzheimer's disease."
answer
"My client is in her 90s, so I don't expect her to respond to the therapy like a 50-year-old does."
question
35. Which of the following statements made by an older adult client best reflects a healthy adjustment to the aging process and its physical limitations? 1. "I use to run in marathons, but now I truly enjoy a 1 mile walk around the park." 2. "I see friends my age just rocking on the porch. Not me; I want to stay physically active." 3. "When I can't get around like I do now, I'll watch TV and catch up on my favorite programs." 4. "I'll miss working in my garden when the arthritis gets bad, but I'll find something else to keep me busy."
answer
"I use to run in marathons, but now I truly enjoy a 1 mile walk around the park."
question
36. Which of the following statements, made by the daughter of an older adult client concerning bring her mother home to live with her family, presents the greatest concern for the nurse? 1. "If this doesn't work out, she can always go to live with my sister." 2. "I don't think she will react very well to me making decisions for her." 3. "I'm afraid that mom will be depressed and really miss her home terribly." 4. "My children will just have to adjust to having their grandmother with us."
answer
"I don't think she will react very well to me making decisions for her."
question
37. A nurse is caring for an older adult client preparing for discharge to a nursing center after having hip surgery. Which of the following nursing responses is most therapeutic when dealing with the client's concern that she, "will never go back home"? 1. "What makes you think that this transfer to the nursing center will be permanent?" 2. "The reason for this transfer is only to support you while you continue to recuperate." 3. "The decision to stay in the nursing center is yours to make. When you want to leave no one will stop you." 4. "The nursing center is a lovely place with a wonderful staff of caring people. Just give it a chance. You may like it."
answer
"What makes you think that this transfer to the nursing center will be permanent?"
question
38. A nurse caring for older adults in an assistive living facility recognizes that a client's quality of life needs are best determined by: 1. Excellent physical, social, and emotional nursing assessments 2. A working knowledge of this age-group's developmental needs 3. A therapeutic nurse-client relationship that facilitates communication 4. The client's ever-changing physical, emotional, and cognitive abilities
answer
A therapeutic nurse-client relationship that facilitates communication
question
39. A nurse is preparing to perform an assessment on an older adult client newly admitted to a nursing center. Which of the following statements made by the nurse best reflects the unique needs of this client regarding the assessment process? 1. "I will be back after you are settled in and we can devote enough time to this assessment process." 2. "We will need to move you to the examination room so that you will be comfortable during the assessment." 3. "I have to perform an assessment as part of the admission process, is this a good time for you to help me with it?" 4. "Since this move has been both physical and emotionally stressful, I will make this assessment as concise and brief as possible."
answer
"I will be back after you are settled in and we can devote enough time to this assessment process."
question
2. The nurse formulates a diagnosis of knowledge deficit related to complications of pregnancy. One outcome criterion is that the client can state five symptoms that indicate a possible problem that should be reported. The client is able to tell the nurse three symptoms. The evaluation statement would be: 1. Goal met; client able to state three symptoms 2. Goal not met; client able to list three symptoms 3. Goal not met; client unable to list five symptoms 4. Goal partially met; client able to state three symptoms
answer
Goal partially met; client able to state three symptoms
question
6. Based on the following outcome criterion determined by the nurse: "Client will independently complete necessary assessments prior to administration of digoxin (cardiotonic)" the nurse will evaluate the client's ability to: 1. Assess the respiratory rate 2. Palpate the radial pulse 3. Review dietary habits 4. Inspect color of the skin
answer
Palpate the radial pulse
question
8. The client has a nursing diagnosis of impaired gas exchange as a result of excessive secretions. An outcome for the client is that the airways will be free of secretions. A positive evaluation will focus upon the client's: 1. Respiratory rate 2. Complaint of chest pain 3. Lungs clear bilaterally on auscultation 4. Ability to perform incentive spirometry
answer
Lungs clear bilaterally on auscultation
question
9. A client shares with the nurse that they have, "almost reached the goal of smoking only one-half pack of cigarettes a day." The best example of a nursing intervention to correct this unmet outcome is: 1. Discuss with the client the desire to comply with the ordered therapy 2. Suggest that the client use another smoking cessation tool to achieve the goal 3. Reevaluate the time frame originally decided upon for achievement of the goal 4. Suggest that the strength of the prescribed nicotine patches be increased to 21 mg
answer
Suggest that the strength of the prescribed nicotine patches be increased to 21 mg
question
10. The primary purpose of the nursing evaluation process is to: 1. Determine the effectiveness of the nursing care provided 2. Identify interventions that are ineffective in achieving client goals 3. Establish the progress the client is making towards health and wellness 4. Critique the nurse's ability to implement appropriate nursing interventions
answer
Determine the effectiveness of the nursing care provided
question
11. Which of the following statements best reflects a goal based on a clinical standard of practice? 1. Client will lose 10 pounds in 90 days. 2. Client will walk 30 feet with minimal assistance. 3. Client's peripheral intravenous site will be free of redness. 4. Client's chronic pain will be managed with oral medication by discharge.
answer
Client's peripheral intravenous site will be free of redness.
question
12. Which of the following outcomes best reflects a nurse-sensitive client outcome? 1. Client will consume 75% of all meals. 2. Client will perform personal hygiene daily. 3. Client will experience no falls during hospitalization. 4. Client will report lessened anxiety regarding surgical procedure.
answer
Client will experience no falls during hospitalization.
question
13. The nurse has identified a nursing diagnosis of knowledge deficit regarding the need to monitor blood glucose levels daily. Which of the following statements best reflects the client's understanding of the need for therapy? 1. Client agrees to test blood glucose levels 4 times a day. 2. Client records blood glucose levels for a 3-week period. 3. Client is observed testing his blood glucose level before breakfast. 4. Client is able to demonstrate the proper technique for performing a finger stick.
answer
Client records blood glucose levels for a 3-week period.
question
14. Which of the following nursing notes demonstrates the best evaluation of nursing interventions regarding the care provided? 1. "Pressure ulcer located on left heel has shown improvement." 2. "Pressure ulcer located on left heel has responded to treatment." 3. "Pressure ulcer on left heel is no longer producing purulent drainage." 4. "Pressure ulcer on left heel has not enlarged in size within the last 24 hours."
answer
"Pressure ulcer on left heel is no longer producing purulent drainage."
question
15. Which of the following statements made by a client's family is the most reliable for use in the evaluation of a client's outcome? 1. "Mom has been eating 90% of all of her meals since she's been home." 2. "My daughter is in much less pain now that she is going to physical therapy." 3. "My husband has been less depressed since he's been on that antidepressant pill." 4. "Mom has been so much better since she's been able to get up and walk by herself."
answer
"Mom has been eating 90% of all of her meals since she's been home."
question
16. A nurse is providing care for a client receiving normal saline when the IV infiltrates. Which of the following nursing actions represents the evaluation phase of the nursing process? 1. IV is discontinued. 2. Warm compress applied to IV site. 3. Site reinspected for presence of swelling. 4. IV site observed as having significant swelling.
answer
Site reinspected for presence of swelling.
question
17. Which of the following questions, asked by a nurse, best reflects an understanding of effective evaluation? 1. "Do you feel confident in the use of your glucometer?" 2. "Have you been following your low carbohydrate diet?" 3. "Any questions regarding the tests you are scheduled for today?" 4. "May we review what we discussed earlier about your medications?"
answer
"May we review what we discussed earlier about your medications?"
question
19. Which of the following statements best defines quality improvement (performance improvement)? 1. The assessment of the delivery system responsible for the implementation of client-oriented interventions 2. Integration of evidence-based practice research into the delivery process used to implement client-oriented interventions 3. High-priority evaluation process directed towards differentiating between good and poor intervention delivery by providers 4. An ongoing evaluation of interventions that is used to improve the delivery of health care for the purpose of managing the client's needs
answer
An ongoing evaluation of interventions that is used to improve the delivery of health care for the purpose of managing the client's needs
question
22. When a client goal is unmet, which of the following nursing actions is most appropriate? 1. Reevaluation of the original client goal 2. Selection of new but appropriate interventions 3. Evaluation of the client's ability and motivation to be compliant 4. Repetition of the entire nursing process regarding the nursing diagnosis
answer
Repetition of the entire nursing process regarding the nursing diagnosis
question
1. Which of the following is a recognized focus area for quality improvement (performance improvement) evaluations? (Select all that apply.) 1. Effective care 2. Delivery of care 3. Client satisfaction 4. Exceeding the standard of care 5. Identification of 'missed' client needs 6. Multidisciplinary approach to client care
answer
1. Effective care 2. Delivery of care 3. Client satisfaction 4. Exceeding the standard of care
question
3. The client is scheduled to receive Coumadin (an anticoagulant) at 9:00 AM. His morning laboratory results show him to have a high partial thromboplastin time (PTT). His nurse decides to withhold the Coumadin. Which step of the implementation process is she using? 1. Reassessing the client 2. Stating an expected outcome 3. Revising the nursing diagnosis 4. Modifying the nursing care plan
answer
Modifying the nursing care plan
question
6. An example of a cognitive nursing skill is: 1. Providing a soothing bed bath 2. Communicating with the client and family 3. Giving an injection to the client per the physician's orders 4. Recognizing the potential complications of a blood transfusion
answer
Recognizing the potential complications of a blood transfusion
question
7. An enterostomal nurse shows a client's significant other how to assist with the supplies for the ostomy and how to manipulate the ostomy equipment. In demonstrating this technique to the client's significant other, the nurse is using what type of nursing skill? 1. Affective 2. Cognitive 3. Interactive 4. Psychomotor
answer
Psychomotor
question
9. The plan of care offers a number of different types of nursing interventions that may be incorporated in. An example of a nurse implemented specific life-saving measure is: 1. Administering analgesics 2. Restraining a violent client 3. Initiating stress-reduction therapy 4. Teaching the client how to take his/her pulse rate
answer
Restraining a violent client
question
10. To provide optimum care, a nursing intervention should be based on: 1. An appropriate nursing diagnosis 2. Subjective and objective client data 3. Sound clinical judgment and knowledge 4. Identified physical and psychosocial needs of the client
answer
Sound clinical judgment and knowledge
question
12. Which of the following interventions best reflects the nurse's understanding of direct care interventions regarding a cognitively impaired client's need for social interaction? 1. Arranging for the client to attend a "sing along" in the dayroom 2. Helping the client place a long distance telephone call to his daughter 3. Turning the client's television on when his or her favorite program is playing 4. Talking about the client's favorite sport's team while redressing his or her wound
answer
Talking about the client's favorite sport's team while redressing his or her wound
question
13. The primary reason for the establishment of standing orders is to: 1. Provide appropriate nursing autonomy in settings where client needs can change rapidly 2. Facilitate adequate care when direct contact with a primary health care provider is not immediately possible 3. Allow nurses to provide certain routine therapies without first notifying the primary health care provider 4. Afford the client interventions that reflect the appropriate standard of care in the absence of a primary health care provider
answer
Provide appropriate nursing autonomy in settings where client needs can change rapidly
question
14. Which of the following statements best reflects the nurse's understanding of the function of client reassessment? 1. "The client's blood pressure is lower this morning than it was yesterday morning." 2. "30 minutes after receiving his pain medication, the client evaluated his pain at 3 out of 10." 3. "Turning the client every 2 hours has helped in the healing of the pressure ulcer on his coccyx." 4. "Since the client has been ambulating to the bedroom without difficulty, I'll walk with him to the dayroom after dinner."
answer
"Since the client has been ambulating to the bedroom without difficulty, I'll walk with him to the dayroom after dinner."
question
15. Which of the following statements made by a nurse practitioner best reflects an understanding of the availability of clinical practice guidelines? 1. "Clinical guidelines are so very helpful in providing the most up-to-date nursing care." 2. "I'm sure we could get a team together and develop a pressure ulcer prevention protocol or search sites for established protocols." 3. "I am particularly impressed by the type 2 diabetic guidelines posted on the National Guidelines Clearinghouse (NGC) site." 4. "I'm told that for gerontological issues, the Gerontological Nursing Interventions Research Center (GNIRC) is the primary resource site."
answer
"I am particularly impressed by the type 2 diabetic guidelines posted on the National Guidelines Clearinghouse (NGC) site."
question
16. The fundamental goal for the development of a protocol for care of a client who has had a myocardial infarction client is to: 1. Implement care that has its basis in evidence-based practice 2. Produce care plans that are specific to the individual client needs 3. Improve the standard of care provided to the clients cared for on that unit 4. Provide the staff on that unit with guidelines to ensure the delivery of quality care
answer
Improve the standard of care provided to the clients cared for on that unit
question
17. Which of the following nursing actions is most likely a result of the nurse's clinical experience? 1. Placing an immobile client on a turning schedule 2. Always assessing a client's IV site before hanging a new bag of fluid 3. Requesting that the nursing assistant have vital signs recorded by 0815 4. Administering a pain medication 30 minutes before changing a burn dressing
answer
Always assessing a client's IV site before hanging a new bag of fluid
question
20. Which of the following statements made by a new graduate nurse regarding the modification of a client's care plan requires immediate follow-up by the nurse's preceptor? 1. "I will review the care plan before I do my charting." 2. "The client prefers to bathe at night, so that's what I'll do." 3. "I gave her a bed bath this morning, but she could really manage showering herself." 4. "The order reads clear liquids, but I hear good bowel sounds and she's really hungry."
answer
"The order reads clear liquids, but I hear good bowel sounds and she's really hungry."
question
21. Which of the following statements regarding utilization of personnel made by a new graduate nurse requires immediate follow-up by the nurse's mentor? 1. "My LPN is really good with dressings, so I usually delegate them to her." 2. "I always take the time to ambulate a post op client the first time out of bed." 3. "I always try to help my nursing assistant with the clients who require a total bed bath." 4. "I have my nursing assistant take and document all vital signs and intake and outputs."
answer
"I have my nursing assistant take and document all vital signs and intake and outputs."
question
3. The nurse writes the following goal for a client who is hypertensive: "Client will maintain a blood pressure within acceptable limits." Which of the following would be the most appropriate outcome criterion? 1. "Client will request pain medication as needed." 2. "Client will experience no headache or dizziness." 3. "Client will identify at least two things that cause stress." 4. "Client will have a 7 AM blood pressure reading less than 140/90."
answer
"Client will have a 7 AM blood pressure reading less than 140/90."
question
6. In order that they are clear and easily understood by other members of the health care team, the nurse recognizes that client goals or outcomes should be documented according to specific criterion. Of the following, the outcome statement that best meets the established criteria is: 1. "Client will describe activity restrictions." 2. "Client will verbalize understanding of treatments." 3. "Client will be ambulated in hallway 3 times each day." 4. "Client's respiratory rate will remain within 20 to 24 breaths per minute by 9/24."
answer
"Client's respiratory rate will remain within 20 to 24 breaths per minute by 9/24."
question
8. The nurse is involved in requesting a management consultation for personnel-related issues. Which of the following is true regarding the consultation process in which the nurse is involved? 1.The problem area should be totally delegated to the consultant. 2. Consultation is often used when the exact problem remains unclear. 3. The problem area is identified by any member of the health care team. 4. Feelings about the problem should be described to the consultant by the nurse.
answer
Consultation is often used when the exact problem remains unclear.
question
10. The nurse recognizes that client goals or outcomes should be documented according to specific criterion in order that they are clear and easily understood by other members of the health care team. Of the following, the outcome statement that best meets the established criteria is the following: 1. "Vital signs will return to within normal levels for a middle aged adult." 2. "Nursing assistant will ambulate the client in the hallway 3 times each day." 3. "Lungs will be clear to auscultation and respiratory rate will be 20/minute." 4. "Output will be at least 100 mL/hour of clear yellow urine within 24 hours."
answer
"Output will be at least 100 mL/hour of clear yellow urine within 24 hours."
question
11. In goal setting, the nurse is aware that the factor that is associated with available client resources and motivation is: 1. Realistic 2. Observable 3. Measurable 4. Client-centered
answer
Realistic
question
12. Nursing interventions may be categorized based upon the degree of nursing autonomy. An example of a nurse-initiated intervention is: 1. Providing client teaching 2. Administering medication 3. Ordering a liver CAT scan 4. Referring a client to physical therapy
answer
Providing client teaching
question
13. Nursing interventions may be categorized based upon the degree of nursing autonomy. Which of the following nursing interventions is considered as physician- or prescriber-initiated? 1. Taking vital signs 2. Providing support to a family 3. Changing a dressing 2 times each day 4. Measuring intake and output each shift
answer
Changing a dressing 2 times each day
question
14. Which one of the following interventions selected by the nurse is classified as Level 2, Domain 2 (Physiological: complex)? 1. Maintaining regular bowel elimination 2. Promoting the health of the entire family 3. Managing severely restricted body movement 4. Restoring tissue integrity to areas damaged by friction
answer
Restoring tissue integrity to areas damaged by friction
question
15. In documentation of nursing care plans, critical pathways differ from traditional nursing care plans in their: 1. Client outcomes 2. Client assessment 3. Nursing interventions 4. Multidisciplinary approach
answer
Multidisciplinary approach
question
16. Nursing interventions should be documented according to specific criteria in order that they may be clearly understood by other members of the nursing team. The most appropriate of the following intervention statements is: 1. Offer fluids to the client q2h 2. Observe the client's respirations 3. Change the client's dressing daily 4. Irrigate the nasogastric tube q2h with 30 ml normal saline
answer
Irrigate the nasogastric tube q2h with 30 ml normal saline
question
17. Nursing interventions should be documented according to specific criteria in order that they may be clearly understood by other members of the nursing team. The most appropriate of the following intervention statements is the following: 1. "Take vital signs." 2. "Refer client to a therapist." 3. "Turn client as needed while in bed." 4. "Apply two 4 × 4 dry gauze dressing pads tid."
answer
"Apply two 4 × 4 dry gauze dressing pads tid."
question
18. Care plans created by nursing students usually differ from those that are completed by nurses working on client units. An aspect of the plan that is usually included in the student's care plan but not in the client's record is: 1. Client outcomes 2. Nursing diagnoses 3. Scientific rationales 4. Nursing interventions
answer
Scientific rationales
question
19. The purpose and distinction of a concept map, which a nurse may use when implementing a plan of care, are for: 1. Multidisciplinary communication 2. Quality assurance in the health care facility 3. Provision of a standardized format for client problems 4. Identification of the relationship of client problems and interventions
answer
Identification of the relationship of client problems and interventions
question
20. A client is newly diagnosed with diabetes mellitus. The nurse identifies a nursing diagnosis of knowledge deficient related to new diagnosis and treatment needs. The most appropriate outcome statement based upon the established criteria is the following: 1. "Client will perform glucose measurements often." 2. "Client will appear less anxious regarding diagnosis." 3. "Urinary output will reach normal young adult levels." 4. "Client will independently perform subcutaneous insulin injection by 8/31."
answer
"Client will independently perform subcutaneous insulin injection by 8/31."
question
21. Which of the following is the best example of an intermediate prioritized client need for a client diagnosed with risk of injury related to poor skin integrity? 1. Applying adequate clothing to ensure the client's warmth 2. Providing sufficient quantities of an aloe-based skin lotion 3. Helping the client select her favorite foods from the menu form 4. Dressing the client's feet in non-skid soled slippers when ambulating
answer
Providing sufficient quantities of an aloe-based skin lotion
question
22. Which of the following would be the best example of a short-term safety goal for a client who recently experienced abdominal surgery? 1. The client will show no systemic or local signs of infection by time of discharge from hospital. 2. The client will demonstrate an understanding of the proper use of patient-controlled analgesia (PCA). 3. The client will demonstrate effective coughing and deep-breathing techniques within 2 hours of surgery. 4. The client will consistently use the call bell to notify the staff of a need for assistance to the bathroom upon return to the nursing unit.
answer
The client will consistently use the call bell to notify the staff of a need for assistance to the bathroom upon return to the nursing unit.
question
23. Which of the following would be the most appropriate outcome criterion for the goal, "Client's pain will be managed to within an acceptable level within 30 minutes of receiving pain medication." 1. Client will deny presence of any pain or discomfort. 2. Client will rate pain at a level of 3 or less out of a possible 10. 3. Client will demonstrate ability to request pain medication as needed. 4. Client will identify two external factors that decrease presence of pain.
answer
Client will rate pain at a level of 3 or less out of a possible 10.
question
24. The nurse is caring for a newly admitted client who is scheduled for diagnostic testing in the morning. Which of the following client needs should take priority? 1. Inventory of clothes and other personal belongings 2. Orientation to the nursing unit and individual room 3. Interview regarding medications currently being taken 4. Assessment of body systems for presurgery checklist
answer
Orientation to the nursing unit and individual room
question
25. Which of the following outcomes, made by a nurse planning care for a client recently fitted with a hearing aid, best reflects an understanding of short-term client education goals? 1. Client will properly clean the hearing aid ear piece daily with soap and water. 2. Client will state 3 positive effects of wearing his hearing aid at follow-up appointment. 3. Client will wear hearing aid while awake to help improve his ability to understand instructions. 4. Client will demonstrate ability to change the batteries in his hearing aid before leaving clinic today.
answer
Client will demonstrate ability to change the batteries in his hearing aid before leaving clinic today.
question
26. Which of the following statements made by a new nursing graduate best reflects an understanding of expected outcomes? 1. "It gives the client something positive to strive towards." 2. "They are statements of how the client's behavior should change." 3. "They are measurable criteria by which I can evaluation whether a goal has been achieved." 4. "They provide the client with suggestions on how to achieve their long and short term goals."
answer
"They are measurable criteria by which I can evaluation whether a goal has been achieved."
question
27. A nurse is caring for a client newly diagnosed with diabetes mellitus. Which of the following statements best reflects an understanding of client-centered goals? 1. "The client's A1C levels will be 7 or below at the first testing date." 2. "The client will experience no blood sugar readings below 60 mg/dL before first follow up visit." 3. "The client will be visited weekly by home health nursing staff beginning 1 week after discharge." 4. "The client will demonstrate the ability to appropriately measure blood sugar levels using a glucometer by discharge from nursing unit."
answer
"The client will demonstrate the ability to appropriately measure blood sugar levels using a glucometer by discharge from nursing unit."
question
28. The expected outcome that best evaluates the presurgical goal of, "Client will understand purpose of coughing and deep breathing within 4 hours of returning to room" is: 1. Client will demonstrate proper technique for coughing and deep breathing 2. Client will cough and deep breathe every 1 hour while awake without staff prompting 3. Client is capable of restating the purpose of coughing and deep breathing in own words 4. Client's lungs will be free of abnormal breath sounds within 1 hour of being returned to room
answer
Client will cough and deep breathe every 1 hour while awake without staff prompting
question
29. Which of the following statements made by the nurse best reflects an understanding of the client's role in goal setting? 1. "He knows what he needs better than anyone else." 2. "When he sets the goals he is more likely to follow the plan." 3. "He identifies the goals and then together we create the plan of action." 4. "He is best suited to determine the level of effort he is capable of providing."
answer
"He is best suited to determine the level of effort he is capable of providing."
question
30. A nurse is caring for a client who experienced short-term memory loss as a result of a head injury. Which of the following statements made by the nurse regarding goal setting requires follow-up by the nurse manager? 1. "The client will certainly need frequent reorientation to the care plan goals." 2. "I will restate the goals I've created for him regularly so as to win his compliance." 3. "I'm not sure that his family will be able to support him with these goals but I will discuss it with them." 4. "He seems very willing to work towards achieving his goals but his condition will certainly create barriers."
answer
"I will restate the goals I've created for him regularly so as to win his compliance."
question
32. Which of the following client-centered goals best rest reflects singular focus? 1. Client will cough and deep breathe every hour while awake. 2. Client will be free of shoulder and elbow pain by discharge. 3. Client will adhere to a low-fat diet and lose 3 pounds in 30 days. 4. Client will ambulate to the bathroom for the purpose of showering daily.
answer
Client will ambulate to the bathroom for the purpose of showering daily.
question
33. The nurse realizes that goals should be singular in focus primarily because: 1. The nurse will find it difficult to modify the plan of care if the goals are not met. 2. The client may not have the strength to accomplish multiply behavioral changes. 3. The client may have difficulty focusing on more than one behavioral modification at a time. 4. The nurse will find it difficult to identify appropriate interventions to address multiple behaviors.
answer
The nurse will find it difficult to modify the plan of care if the goals are not met.
question
34. Which of the following goals concerning client anxiety is the best example of measurability? 1. Client will be less anxious by discharge. 2. Client will appear less anxious by discharge. 3. Client will report anxiety at less than 3 out of 5 by discharge. 4. Client pulse rate and blood pressure will be within normal limits by discharge.
answer
Client will report anxiety at less than 3 out of 5 by discharge.
question
35. Which of the following goals best reflects measurability? 1. Client's emotional state will be stable by time of discharge. 2. Client will experience normal sensations in feet by discharge. 3. Client will report being free of shoulder pain by discharge. 4. Client will have acceptable range of motion in elbow by discharge.
answer
Client will report being free of shoulder pain by discharge.
question
36. When developing appropriate nurse-initiated interventions for a client admitted to an acute care facility for abdominal pain, the nurse must first consider: 1. The institution's policies and procedures 2. The state's defined scope of nursing practice 3. The client's physiological and psychological needs 4. The scientific rationale for the proposed nursing action
answer
The state's defined scope of nursing practice
question
37. The nurse realizes that the primary nursing responsibility regarding a physician-initiated intervention is to: 1. Facilitate the intervention in a timely manner 2. Evaluate the client's response to the intervention 3. Possess the technical skills required to implement the intervention 4. Provide client education regarding the implementation of the intervention
answer
Possess the technical skills required to implement the intervention
question
38. The primary function of a care plan is to provide: 1. The client with continuity of care 2. The staff with written client-centered nursing interventions 3. An established criteria for the evaluation of nursing outcomes 4. An organized means of exchanging information between caregivers
answer
The client with continuity of care
question
1. Which of the following characteristics are considered guidelines for the writing of appropriate goals and outcomes? (Select all that apply.) 1. Singular 2. Realistic 3. Practical 4. Observable 5. Measurable 6. Meaningful
answer
1. Singular 2. Realistic 4. Observable 5. Measurable
question
1. The nurse uses nursing diagnoses after completion of the client assessment, because they: 1. Are required for accreditation purposes 2. Identify the domain and focus of nursing 3. Assist the nurse to distinguish medical from nursing problems 4. Make all client problems become more quickly and easily resolved
answer
Identify the domain and focus of nursing
question
2. A 53-year-old client is seen at the clinic for a yearly physical examination. In evaluating the client's weight, the nurse also considers the age and height. This is an example of: 1. Defining the client problem 2. Recognizing gaps in data assessment 3. Comparing data with normal health patterns 4. Drawing conclusions about the client's response
answer
Comparing data with normal health patterns
question
3. Of the following statements, which one is an example of an appropriately written nursing diagnosis? 1. Acute pain related to left mastectomy 2. Impaired gas exchange related to altered blood gases 3. Deficient knowledge related to need for cardiac catheterization 4. Need for high protein diet related to alteration in client nutrition
answer
Deficient knowledge related to need for cardiac catheterization
question
4. Of the following statements, which one is an example of an appropriately written nursing diagnosis? 1. Risk for change in body image related to cancer 2. Cardiac output decreased related to motor vehicle accident 3. Ineffective airway clearance related to increased secretions 4. Potential for injury related to improper teaching in the use of crutches
answer
Ineffective airway clearance related to increased secretions
question
5. The nurse has diagnosed the client's problem as altered elimination. From the database the nurse identifies all the following as appropriate etiologies for this diagnosis except: 1. Poor fiber intake 2. Limited fluid intake 3. Total hip replacement 4. Lower abdominal discomfort
answer
Total hip replacement
question
7. The nurse recognizes that which one of the following statements is true with regard to the formulation of nursing diagnoses? 1. The diagnosis should identify a "cause and effect" relationship. 2. The diagnosis must remain constant during the client's hospitalization. 3. The etiology of the diagnosis must be within the scope of the health care team's practice. 4. The diagnosis should include the problem and the related contributing conditions.
answer
The diagnosis should include the problem and the related contributing conditions.
question
12. Which of the following is an appropriate etiology for a nursing diagnosis? 1. Incisional pain 2. Poor hygienic practices 3. Need to offer bedpan frequently 4. Inadequate prescription of medication
answer
Incisional pain
question
13. Of the following statements, which one is an example of an appropriately written nursing diagnosis? 1. Diarrhea related to food intolerance 2. Alteration in comfort related to pain 3. Risk for impaired skin integrity related to poor hygiene habits 4. Potential complications related to insufficient vascular access
answer
Diarrhea related to food intolerance
question
14. Of the following statements, which one is an example of an appropriately written nursing diagnosis? 1. Anxiety related to cardiac monitor 2. Pain related to difficulty ambulating 3. Chronic pain related to insufficient use of medication 4. Bedpan required frequently as a result of altered elimination pattern
answer
Chronic pain related to insufficient use of medication
question
15. Based on the following information, what would the nurse identify as the most appropriate nursing diagnosis? The client has abnormal breath sounds, dyspnea, an intermittent cough, and variable respiratory rate. 1. Risk for injury 2. Excess fluid volume 3. Ineffective airway clearance 4. Impaired spontaneous ventilation
answer
Ineffective airway clearance
question
16. Which one of the following is a NANDA International nursing diagnosis label? 1. Frequent urination 2. Coughing and dyspnea 3. Risk for impaired parenting 4. Abnormal hygienic care practices
answer
Risk for impaired parenting
question
17. When asked to define "Nursing Diagnosis" the nurse's best response is: 1. "It is the second step in the Nursing Process." 2. "It is the process of defining a client's problems." 3. "It correlates a client's problem with a condition a nurse is competent to treat." 4. "It focuses care a licensed nurse can provide with the identified needs of a client."
answer
"It correlates a client's problem with a condition a nurse is competent to treat."
question
18. The nurse's initial responsibility in the management of a client's collaborative problem is to: 1. Monitor for changes 2. Advocate for the client 3. Implement interventions 4. Evaluate client outcomes
answer
Monitor for changes
question
19. The nurse has identified deficient knowledge regarding surgery for a client who is scheduled for an outpatient procedure. Which of the following instructional topics will best minimize the client's anxiety regarding the procedure? 1. Assure the client that preoperative sedation will be administered. 2. Discuss the pre- and postprocedure care that will be provided. 3. Provide a detailed explanation of why the procedure is necessary. 4. Guarantee that family will be regularly updated during the procedure.
answer
Discuss the pre- and postprocedure care that will be provided.
question
20. The nursing diagnosis of acute pain falls under which of the following comfort domain classifications? 1. Social comfort 2. Physical comfort 3. Interpersonal comfort 4. Environmental comfort
answer
Physical comfort
question
21. When asked to define the purpose of diagnostic reasoning, the best nursing response is: 1. "Diagnostic reasoning is the foundation of the second step of the nursing process; Nursing Diagnosis." 2. "The diagnostic reasoning process flows from the assessment process and includes decision-making steps." 3. "Diagnostic reasoning includes data clustering, identifying client needs and formulating the diagnosis or problem." 4. "Diagnostic reasoning involves using the assessment collected on a specific client to logically arrive at an appropriate nursing diagnosis."
answer
"Diagnostic reasoning involves using the assessment collected on a specific client to logically arrive at an appropriate nursing diagnosis."
question
22. A nursing student expresses some confusion about identifying the appropriate nursing diagnosis for a specific client. Which of the following responses by the clinical instructor is most instructional? 1. "After defining the client's symptomatology, eliminate those nursing diagnoses that are not supported by the database." 2. "Assess your client and then select the nursing diagnosis that has the greatest number of observable defining characteristics." 3. "After assessing the client, compare their symptoms carefully to the defining characteristic of the nursing diagnosis in order to support or eliminate it as applicable." 4. "With experience you will become skilled at identifying the defining characteristics of a nursing diagnosis in your client. Until that time use a nursing diagnosis book to help in the selection process."
answer
"After assessing the client, compare their symptoms carefully to the defining characteristic of the nursing diagnosis in order to support or eliminate it as applicable."
question
23. A client newly diagnosed with type 2 diabetes mellitus asks the nurse to explain, "what the diagnosis means." Which of the following rationales best supports the nurse's determination that the client has knowledge deficit rather than a readiness for enhanced knowledge? 1. The client initiated the question. 2. This is a new diagnosis for the client. 3. The client identified a lack of understanding. 4. Type 2 diabetes mellitus is a complicated disease process.
answer
This is a new diagnosis for the client.
question
24. Which of the following responses best reflects an understanding of the purpose of the "related to" phrase attached to the diagnostic label deficient knowledge regarding postoperative routines? 1. "To focus on the cause of the client's needs" 2. "To identify the etiology of the client's diagnosis" 3. "To provide for individualization of the nursing interventions" 4. "To communicate the client's deficits to the nursing staff"
answer
"To provide for individualization of the nursing interventions"
question
25. Which of the following assessment findings best supports the nursing diagnosis of pain in right knee joint related to degenerative process? 1. Paternal family history of osteoarthritis has been reported. 2. Client is observed grimacing when walking to bathroom. 3. Right knee appears edematous when compared to left knee. 4. Client rated the pain felt after walking at a 6 on a scale of 1 to 10.
answer
Client is observed grimacing when walking to bathroom.
question
26. Which of the following statements made by a nursing student regarding the cultural characteristics of pain requires immediate follow-up by the clinical instructor? 1. "I can tell when my Hispanic clients are in pain." 2. "Moaning is a classic sign of pain in most cultures." 3. "All clients will tell you when they need pain medication." 4. "Chronic pain is difficult to manage especially for the stoic individual."
answer
"All clients will tell you when they need pain medication."
question
27. Which of the following statements best reflects the nurse's understanding of the primary nursing-related purpose of a concept map? 1. To facilitate holistic nursing care 2. To provide visualization of the client's health problems 3. To assist in the identification of client-oriented nursing diagnoses 4. To demonstrate the relationship between the client's various health problems
answer
To demonstrate the relationship between the client's various health problems
question
28. Which of the following statements made by the nurse reflects the best understanding of the usefulness of a concept map to client care? 1. "Concept maps help me see the whole client, not just individual health problems." 2. "Concept maps can be easily edited to reflect a client's ever changing health needs." 3. "I need help organizing my assessment data and concept mapping is really good for that." 4. "I like concept mapping because it helps me focus on how the disease processes affect the client."
answer
"Concept maps help me see the whole client, not just individual health problems."
question
29. A client expresses concern over a scheduled intravenous pyelogram by stating, "I don't know what to expect." Which of the following nursing diagnoses is most appropriate for this client need? 1. Anxiety related to scheduled diagnostic testing 2. Knowledge deficit regarding need for diagnostic testing 3. Knowledge deficit related to need for intravenous pyelogram 4. Anxiety related to lack of knowledge concerning intravenous pyelogram
answer
Anxiety related to lack of knowledge concerning intravenous pyelogram
question
1. Research has shown that which of the following nursing skills is best strengthened through the use of concept mapping? (Select all that apply.) 1. Client teaching related to health and wellness topics 2. Evaluation of client outcomes in regards to nursing care 3. Identification of patterns in the client's health assessment data 4. Recognition of relationships among the client's various health issues 5. Planning specialized nursing interventions to meet a client's health needs 6. Facilitating assessment data collection through observation and communication
answer
2. Evaluation of client outcomes in regards to nursing care 3. Identification of patterns in the client's health assessment data 4. Recognition of relationships among the client's various health issues 5. Planning specialized nursing interventions to meet a client's health needs
question
1. A client interview consists of three phases. The nurse recognizes that those phases are: 1. Orientation, working, termination 2. Introduction, controlling, selection 3. Introduction, assessment, conclusion 4. Orientation, documentation, database
answer
Orientation, working, termination
question
2. During the admission history, the client states that he has trouble breathing at night. In obtaining data for a problem-oriented database, the nurse should first question the client about: 1. The onset and duration of his present breathing problem 2. His personal smoking, alcohol use, and exercise practices 3. Any extended family members who have diagnosed heart disease 4. Changes in other body systems that the client perceives as problematic
answer
The onset and duration of his present breathing problem
question
3. The nurse begins the assessment of a client that has come to the emergency department experiencing chest pain by asking the client about: 1. A family history of heart problems 2. Medications currently being taken at home 3. Questions or concerns about hospitalization 4. The onset, severity, and duration of the chest pain
answer
The onset, severity, and duration of the chest pain
question
4. A nurse seeks to organize the data obtained from the client in a logical manner. The organizational method that identifies relationships between factors and symptoms in the database is known as: 1. Clustering data 2. Validating data 3. Peer reviewing 4. Problem statement
answer
Clustering data
question
5. The client recently became febrile and stated he "felt hot." The nurse takes the client's temperature and finds it to be 38.2° C. In addition, the pulse rate is 88 beats per minute, and his blood pressure is 168/80 mm Hg. Which of the following is an example of subjective data? 1. Pulse rate of 88 beats per minute 2. Blood pressure of 168/80 mm Hg 3. The statement regarding his feeling hot 4. The supported fact that he became febrile
answer
The statement regarding his feeling hot
question
6. The nurse decides to interview the client using the open-ended question technique. Which of the following statements reflects this type of questioning? 1. "Is your pain worse or better than it was an hour ago?" 2. "Do you believe that your nausea is from the new antibiotic?" 3. "What do you think has been causing your current depression?" 4. "What have you done to alleviate the side effects from your medications?"
answer
"What do you think has been causing your current depression?"
question
7. The nurse is gathering a nursing health history on the client. The client tells the nurse that he just lost his job. Job loss best fits into which of the following categories? 1. Family history 2. Psychosocial history 3. Biographical history 4. Environmental history
answer
Psychosocial history
question
8. The nurse is going to perform the admission history for a newly admitted client on the medical unit. The optimum time for completion of the history is planned for: 1. Coordination with the physician's visit 2. The time when the client's family are visiting 3. Immediately before the client's scheduled MRI testing 4. After the client has become comfortably oriented to the room
answer
After the client has become comfortably oriented to the room
question
9. The nurse has completed an assessment and found that the client has "an activity and exercise abnormality." This type of wording indicates that which of the following organizing formats has been used? 1. Review of systems 2. Nursing health history 3. Gordon's functional health patterns 4. Biographical information database
answer
Gordon's functional health patterns
question
10. After visiting with the client, the nurse documents the assessment data. Both objective and subjective information has been obtained during the assessment. Which of the following is classified as objective data? 1. Pain in the left leg 2. Elevated blood pressure 3. Fear of impending surgery 4. Discomfort upon breathing
answer
Elevated blood pressure
question
11. The primary source of information when completing an assessment of a client that is alert and oriented as he is admitted to the medical center for diagnostic testing is the: 1. Client 2. Physician 3. Family member 4. Experienced unit nurse
answer
Client
question
12. The process of data collection should begin with the nurse performing a: 1. Physical exam 2. Client interview 3. Review of medical records 4. Discussion with other health team members
answer
Client interview
question
13. During an interview, the nurse needs to obtain specific information about the signs and symptoms of the client's health problem. To obtain these data most efficiently, the nurse should use: 1. Channeling 2. Open-ended questions 3. Closed-ended questions 4. Problem-seeking responses
answer
Closed-ended questions
question
14. The nurse is conducting an interview with the client and wants to clarify information that the client has shared. Which response by the nurse is an example of the clarifying technique of communication? 1. "I understand how you must feel." 2. "This medication is used to lower your blood pressure." 3. "You appear anxious. You're wringing your hands constantly." 4. "Could you give me an example of how you handle stressors?"
answer
"Could you give me an example of how you handle stressors?"
question
15. When clustering data according to functional health patterns, the nurse determines that the client is only able to ambulate short distances without becoming fatigued and requires rest periods during morning care. The health pattern that requires intervention is identified by the nurse as: 1. Respiratory 2. Activity and exercise 3. Sleep and rest pattern 4. Self-care deficit: activities of daily living
answer
Activity and exercise
question
17. An ER nurse is interviewing a client who complains of abdominal pain. Which of the following questions asked by the nurse has priority at this time? 1. "Can you describe your pain?" 2. "Have you had this problem before?" 3. "What have you done to ease the pain?" 4. "When did your abdominal pain begin?"
answer
"When did your abdominal pain begin?"
question
20. Which of the following statements best reflects the nurse's correct understanding of the importance of selecting the optimum time for interviewing a client newly admitted to the unit? 1. "I'm going to do the client's history before his family leaves so they can help with the admission history questions." 2. "You are scheduled for some x-rays, so I'd like to complete this admission history interview before you have to leave." 3. "I have some questions to ask you regarding your admission history. I'll be back once you are settled in and comfortable." 4. "Please let me know when the blood lab is finished with the new client so I can complete his admission history interview."
answer
"I have some questions to ask you regarding your admission history. I'll be back once you are settled in and comfortable."
question
21. The nurse is conducting an admissions history interview with a client who has a history of gastroesophageal reflux disease (GERD). Which of the following questions shows the best example of relevant questioning by the nurse? 1. "How long have you been dealing with GERD?" 2. "Are you currently taking any medications for your GERD?" 3. "Do you follow a particular diet to help manage your GERD?" 4. "Do you have any other gastrointestinal problems besides GERD?"
answer
"Do you have any other gastrointestinal problems besides GERD?"
question
22. A new graduate nurse missed cues regarding the client's emotional state at the time of admission. The most therapeutic response to the nurse by her mentor is: 1. "That is why we perform assessments at least daily; so we can catch missed cues." 2. "Everyone has missed cues; don't be too hard on yourself and just keep trying." 3. "You will be less likely to miss client cues as you acquire more experience with assessments." 4. "The positive side to making this mistake is that you won't miss those cues again in another client."
answer
"You will be less likely to miss client cues as you acquire more experience with assessments."
question
23. The nurse is performing a problem-focused assessment when the client reports pain in his left shoulder. Which of the following nursing questions has priority when determining the nature of the pain? 1. "What makes the pain worse?" 2. "When did you first notice the pain?" 3. "What do you do to lessen the pain?" 4. "Can you rate your pain using the pain scale that we've discussed?"
answer
"Can you rate your pain using the pain scale that we've discussed?"
question
24. When following up on a client's report of hip pain during an admission assessment, the most nursing conclusive observation would be: 1. The client tearing when being ambulated to the chair 2. A report from the ancillary staff that the client is reporting pain 3. The client observed grimacing when positioning self in the bed 4. Overhearing the client discuss hip pain with family on the phone
answer
The client observed grimacing when positioning self in the bed
question
25. When obtaining subjective assessment data, the nurse recognizes which of the following client scenarios as being the most likely to produce accurate, credible information? 1. A 50-year-old in the ED reporting chest pain 2. A 70-year-old admitted with fever of unknown origin 3. A 81-year-old receiving follow-up treatment for a hip replacement 4. A 22-year-old being treated at a clinic for a sexually transmitted disease
answer
A 81-year-old receiving follow-up treatment for a hip replacement
question
26. A nurse is observed conducting an assessment interview for a newly admitted client. Which of the following would require immediate follow-up by the nurse's mentor? 1. Conducting the interview with the client's boyfriend present 2. Stopping the interview to answer a page from the nursing station 3. Frequently checking the time while waiting for the client to answer 4. Heard asking the client, "Am I correct; you've rated your pain a 9 out of 10?"
answer
Frequently checking the time while waiting for the client to answer
question
27. Which of the following assessment data provided by a client's family will have the greatest impact on the client's care while hospitalized? 1. "Mom falls asleep fastest with the television on." 2. "Dad starts off the day with hot coffee; it regulates his bowels." 3. "My wife's sister died 4 months ago, and she is still grieving over her loss." 4. "My husband doesn't like to let people know his arthritis is bothering him."
answer
"My husband doesn't like to let people know his arthritis is bothering him."
question
28. What is the most appropriate method for the nurse to communicate a client's wishes to the nurses on the next shift? 1. Document the request in the nursing notes. 2. Include the client's request in the shift report. 3. Place instructions regarding the client's wishes above the client's bed. 4. Verbally inform the unit clerk of the client's request.
answer
Include the client's request in the shift report.
question
29. While discussing a client's medication history, the client tells the nurse that she thinks she is allergic to a particular type of medication. Which of the following nursing actions has priority in this situation? 1. Note the allergy on the client's Kardex. 2. Inform the provider of the client's possible allergy. 3. Review the client's medical record for confirmation of the allergy. 4. Tell the client to have all medications identified before taking them.
answer
Review the client's medical record for confirmation of the allergy.
question
30. The nurse realizes that in order to share information from a client's medical record with another facility, the client must provide written consent. The primary reason for this requirement is to: 1. Facilitate the exchange of information between appropriate parties 2. Minimize the opportunity for this information to be assessed inappropriately 3. Ensure the client's right to have his medical information regarded as personal and confidential 4. Guarantee that the information will be shared with only those requiring it for client care purposes
answer
Ensure the client's right to have his medical information regarded as personal and confidential
question
31. The nurse recognizes that a client's hearing deficits impact the development of the nurse-client relationship. Which of the following has the greatest impact on minimizing this obstacle? 1. Speaking slowly, clearly, and in a normal tone 2. Using various forms of nonverbal communication 3. Relying heavily on touch to convey caring and interest 4. Involving family in discussions concerning meeting client's needs
answer
Using various forms of nonverbal communication
question
32. Which of the following questions will provide the nurse with the best understanding of a terminally ill client's spiritual needs? 1. "Do you have a religious preference?" 2. "Have you given thought to your spiritual needs?" 3. "Is there a particular clergy you would like to visit with?" 4. "Are there any spiritual needs you have that I may help with?"
answer
"Are there any spiritual needs you have that I may help with?"
question
1. Which of the following statements made by the nurse should be included in the orientation phase of a nursing interview? (Select all that apply.) 1. "You're answers will be kept confidential." 2. "My name is Susan Smith and I'm a registered nurse." 3. "We are here to make your hospitalization as pleasant as possible." 4. "I need to ask you some questions that will help with planning your care." 5. "Only those directly involved in your care will have access to this information." 6. "If there is anything you need or help you require simply use your call bell and someone will be right in."
answer
1. "You're answers will be kept confidential." 2. "My name is Susan Smith and I'm a registered nurse." 4. "I need to ask you some questions that will help with planning your care." 5. "Only those directly involved in your care will have access to this information."
question
2. The nurse has determined that the assessment data have resulted in a strong inference that the client is suffering from depression. Which of the following client responses to nursing questions best supports the possibility of depression? (Select all that apply.) 1. "My work environment would depress anyone." 2. "It seems like almost anything can make me cry." 3. "Being here away from my family makes me sad." 4. "I just can't seem to get excited about anything anymore." 5. "The family always thought that my father was depressed." 6. "I like winter because I can just cover up on the couch and sleep."
answer
4. "I just can't seem to get excited about anything anymore." 5. "The family always thought that my father was depressed."
question
3. The goal of the orientation phase of a nursing interview is to: (select all that apply) 1. Initiate the nurse-client relationship 2. Begin identifying the client's needs 3. Earn the trust and confidence of the client 4. Assume the decision role for the client 5. Welcome the client to the nursing unit 6. Gather the client's demographic information
answer
1. Initiate the nurse-client relationship 2. Begin identifying the client's needs 3. Earn the trust and confidence of the client
question
1. Which of the following best reflects the philosophy of critical thinking as taught by a nurse educator to a nursing student? 1. "Think about several interventions that you could use with this client." 2. "Don't draw subjective inferences about your client—be more objective." 3. "Please think harder—there is a single solution for which I am looking." 4. "Trust your feelings—don't be concerned about trying to find a rationale to support your decision."
answer
"Think about several interventions that you could use with this client."
question
2. The second component of critical thinking in the "critical thinking model" is: 1. Experience 2. Competencies 3. Specific knowledge 4. Diagnostic reasoning
answer
Experience
question
3. The nurse enters the room of a client who has a history of heart disease. On looking at the client, the nurse feels that something is "not right" with the client and proceeds to take the vital signs. This is the nurse acting on: 1. Intuition 2. Reflection 3. Knowledge 4. Scientific methodology
answer
Intuition
question
4. The nurse manager has developed a staff protocol for peer evaluation. The nurses on her surgical unit are nervous about using her instrument. If the nurse manager continues to implement the new strategy, which of the following critical thinking attitudes is she portraying? 1. Humility 2. Risk-taking 3. Accountability 4. Independent thinking
answer
Risk-taking
question
5. The nurse is working with a client who has recently had a colostomy and is having difficulty using the provided supplies. The nurse works with the client to see which alternative supplies are easier for the client to use. This is an example of the critical thinking strategy of: 1. Inference 2. Management 3. Problem-solving 4. Diagnostic reasoning
answer
Problem-solving
question
6. Which of the following is an example of a nurse's statement that reflects using the scientific method in the nursing process? 1. "I believe that this client is getting depressed." 2. "The client doesn't look right to me; I think something is wrong." 3. "The client's husband told me that she is feeling very uncomfortable." 4. "The client reports more pain than yesterday and her blood pressure is elevated."
answer
"The client reports more pain than yesterday and her blood pressure is elevated."
question
7. The nurse decides to administer tablets of Tylenol instead of the intramuscular Demerol she has previously been providing her orthopedic client. Which step of the nursing process does this address? 1. Assessment 2. Nursing diagnosis 3. Planning 4. Implementation
answer
Implementation
question
8. The nurse has a multiple client assignment on the surgical unit. On beginning the shift, the nurse needs to determine which postoperative client should be seen first. Of the following, the nurse should go to see the client who: 1. Has a documented blood pressure of 90/50 2. Was medicated for back pain 10 minutes ago 3. Has an order to be out of bed and ambulated 4. Requires instructions for wound care before discharge
answer
Has a documented blood pressure of 90/50
question
9. There are a variety of levels of critical thinking. An example of critical thinking at the complex level is: 1. Giving medication at the time ordered 2. Following a procedure for catheterization step-by-step 3. Reviewing all clients' medical records thoroughly 4. Discussing various alternative pain management techniques
answer
Discussing various alternative pain management techniques
question
10. The nurse is deciding on the type of dressing to use for a client. Which step of the decision-making process is being used when the nurse observes the absorbency of different dressing brands? 1. Defining the problem 2. Making final decisions 3. Testing possible options 4. Considering consequences
answer
Testing possible options
question
11. Which one of the following examples demonstrates the critical thinking attitude of responsibility and authority? 1. Reporting client difficulties 2. Offering an alternative approach 3. Looking for a different treatment option 4. Sharing ideas about nursing interventions
answer
Reporting client difficulties
question
12. Use of the intellectual standard of critical thinking implies that the nurse: 1. Questions the physician's order 2. Recognizes conflicts of interest 3. Listens to both sides of the story 4. Approaches assessment logically
answer
Approaches assessment logically
question
13. A client requires urinary catheterization but has difficulty keeping her legs in the usual position needed for this procedure. The nurse has worked for many years and adapts the procedure to allow the client to lie on her side. This action is based on the critical thinking element of: 1. Curiosity 2. Experience 3. Perseverance 4. Scientific knowledge
answer
Experience
question
14. Which of the following statements made by a nursing student concerning the use of critical thinking and client care requires follow-up by the nursing instructor? 1. "I feel it's good practice to always have alternative interventions in mind." 2. "I trust my feelings about a client's needs since I work hard at knowing my client." 3. "I always try to keep an open mind about what interventions my client will require." 4. "I will wait until my assessment is completed before determining the client's needs."
answer
"I trust my feelings about a client's needs since I work hard at knowing my client."
question
15. Which of the following is the best example of a nurse's use of reflection? 1. The nurse places a client experiencing respiratory difficulties in a high-Fowler's position. 2. The nurse calls the provider when a client reports feeling "chilled and achy" while having an oral temperature of 100.2° F. 3. While caring for a client with a history of asthma, the nurse assesses the client's pulse oximetry reading when he "doesn't sound right." 4. A nurse tells a client; "When you refused to go to physical therapy earlier today I believe you were upset about something else besides the appointment time."
answer
A nurse tells a client; "When you refused to go to physical therapy earlier today I believe you were upset about something else besides the appointment time."
question
16. Which of the following nursing situations best reflects accountability? 1. The nurse takes the oncology nursing certification examination. 2. The nurse files an incident report regarding a medication error. 3. The nurse assesses the client for the possible cause of his pain. 4. The nurse tells the client, "I don't know but I will find out for you."
answer
The nurse files an incident report regarding a medication error.
question
17. Which of the following nursing actions is the best example of problem solving? 1. Requesting the IV team to start an antibiotic drip on a client with a history of being a difficult stick 2. Offering to call the kitchen to provide an alternate breakfast for a client who does not like cooked cereal 3. Trying several difficult wound dressings to determine which one the client can apply the most effectively 4. Calling for another pain medication order when the current drug results in the client experiencing nausea
answer
Trying several difficult wound dressings to determine which one the client can apply the most effectively
question
18. Which of the following clients should be prioritized with the most urgent need for a nursing assessment? 1. A new admission admitted for swelling in the right ankle and knee 2. A second day postoperative client who received pain medication 30 minutes ago 3. A client who the nursing assistant found crying in the bathroom 4. A client ready for discharge who requires a final assessment and documentation
answer
A client who the nursing assistant found crying in the bathroom
question
19. Which of the following nursing interventions is the best example of the implementation step of the nursing process? 1. Determining that the client's ankle edema is worse after he ambulates 2. Asking the client to rate his ankle pain after receiving oral pain medication 3. Arranging for the client to receive pain medication 30 minutes before his ordered ambulation 4. Crushing the client's pain medication to facilitate easier swallowing and thus minimize the risk of choking
answer
Crushing the client's pain medication to facilitate easier swallowing and thus minimize the risk of choking
question
20. Which of the following nursing actions best reflects the consequence stage of the decision-making process? 1. Being physically present when a client is given the results of a tissue biopsy 2. Witnessing the client sign consent for surgery forms before cardiac surgery 3. The client is informed of the various treatments available for his condition. 4. The nurse explains to the client the risks of leaving the hospital against medical advice.
answer
The nurse explains to the client the risks of leaving the hospital against medical advice.
question
21. The concept of nursing responsibility is best reflected in which of the following nursing actions? 1. Providing accurate and timely documentation regarding an incident resulting in a client fall 2. Suggesting that a client might prefer taking a particular medication at bedtime instead of in the morning 3. Posting a note on the unit Kardex how to best apply a dressing to a skin wound on a particular client 4. Referring to the institution's policy manual when unsure of how to handle a client's complaint regarding a social services consult
answer
Referring to the institution's policy manual when unsure of how to handle a client's complaint regarding a social services consult
question
22. Which of the following situations is the best example of a nurse using intellectual standards as a critical thinking tool? 1. Performing a head-to-toe assessment on a new admission 2. Placing a client experiencing shortness of breath on oxygen 3. Arbitrating a complaint between roommates over the television 4. Notifying a provider of a client's allergy to an ordered medication
answer
Placing a client experiencing shortness of breath on oxygen
question
23. The nurse is best demonstrating perseverance by: 1. Having a perfect attendance record 2. Completing a lengthy course on current chemotherapies 3. Repeatedly irrigating the nasogastric tube until it is patent 4. Sitting with a client until she is ready to discuss why she is crying
answer
Sitting with a client until she is ready to discuss why she is crying
question
24. With regards to client care, the most likely reason that a veteran nurse tends to be a more skillful critical thinker than a new graduate nurse is because: 1. The veteran nurse has a varied history of client care experiences 2. Critical thinking improves with experience, longevity, and interest 3. Today's short hospital stays minimize the opportunity to develop critical thinking skills 4. New graduates often lack the self-confidence to take the risks often required of critical decision making
answer
Critical thinking improves with experience, longevity, and interest
question
25. The primary factor that distinguishes a professional nurse's care from care provided by ancillary nursing staff is: 1. Critical thinking 2. Years of education 3. Professional licensure 4. Complexity of the task
answer
Critical thinking
question
26. A clinical nursing instructor asks the nursing students to describe a critical thinker. Which of the following represents the best response? 1. "A person with the educational background to solve problems." 2. "A person who finds the problem and does what is best to fix it." 3. "It's someone who uses the scientific method to solve problems." 4. "Someone who uses a system to work through and solve a problem."
answer
"A person who finds the problem and does what is best to fix it."
question
27. Which of the following statements made by a new graduate nurse regarding a client's care needs requires follow-up by the mentor? 1. "No one really enjoys being hospitalized." 2. "Every client is offered a back rub at bedtime." 3. "All post surgery clients are reluctant to ambulate." 4. "I always spend extra time with new clients to help them relax."
answer
"All post surgery clients are reluctant to ambulate."
question
28. A nurse is caring for an immobile client with a large pressure ulcer on her left ankle. Which of the following statements by the nurse best reflects critical thinking regarding client care? 1. "I'm sure that friction and pressure have caused this problem." 2. "Please be sure that her ankles are well padded when you place her in bed." 3. "Do you have any suggestions on how we can minimize the pressure to her ankles?" 4. "It was an ineffective turning schedule that allowed this to happen so now we will reposition every hour."
answer
"Do you have any suggestions on how we can minimize the pressure to her ankles?"
question
29. A nurse is caring for a 35-year-old client who is 12 hours post mastectomy. The care assistant reports that the client is crying. Which of the following responses by the nurse best reflects the use of analysis regarding this client's care needs? 1. "That surgery is painful. I'll get her pain medication ready." 2. "She was sleeping when I checked 15 minutes ago. I'll go back in right now." 3. "I'll be responsible for her PM care so I can spend some uninterrupted time with her." 4. "A mastectomy is a blow to a woman's self image. I'll notify her provider that she is depressed."
answer
"She was sleeping when I checked 15 minutes ago. I'll go back in right now."
question
30. Which of the following statements made by a nurse regarding personal reflection related to client care requires follow-up by the unit's nurse manager? 1. "Mary and I were comparing foot wound dressing techniques." 2. "I've been caring for orthopedic clients for 10 years and I think I've seen it all." 3. "I can't believe that my client isn't improving after 2 weeks of physical therapy." 4. "I always wean my orthopedic surgery clients onto oral pain medication on postoperative day 4."
answer
"I always wean my orthopedic surgery clients onto oral pain medication on postoperative day 4."
question
1. The scope of a client's health problem is a result of which of the following factors? (Select all that apply.) 1. Religious beliefs 2. Life experiences 3. Lifestyle choices 4. Work environment 5. Family relationships 6. Educational background
answer
2. Life experiences 3. Lifestyle choices 4. Work environment 5. Family relationships
question
1. Which of the following research approaches is an example of an exploratory type of research? 1. Establishing facts and relationships of past events 2. Testing how well a program, practice, or policy is working 3. Refining a hypothesis on the relationships among phenomena 4. Portraying the characteristics of persons, situations, or groups
answer
Refining a hypothesis on the relationships among phenomena
question
2. The Health Information Portability and Accountability Act (HIPAA), implemented in 2003, may influence nursing research in the area of: 1. The cost of the study 2. Where the study may be published 3. What type of study may be conducted 4. How the data will be obtained and protected
answer
How the data will be obtained and protected
question
3. The expected research role for the baccalaureate-prepared nurse is to: 1. Assume the role of a clinical expert 2. Acquire funding for research projects 3. Identify clinical nursing problems in practice 4. Develop methods of inquiry relevant to nursing
answer
Identify clinical nursing problems in practice
question
4. When a nurse researcher distributes an explanatory information sheet to subjects solicited for participation in her study, which of the following ethical principles that guide research is this researcher using? 1. Informed consent 2. Freedom from harm 3. Protection of subjects 4. Confidentiality of subjects
answer
Informed consent
question
5. The nurse takes on ethical responsibilities when conducting research with human subjects. Which of the following violates an ethical responsibility associated with informed consent? 1. Adhering to verbal and written agreements 2. Using data obtained before the initiation of the study 3. Explaining the possibility of unknown risks when appropriate 4. Providing alternatives, including the right of refusal and standard practices
answer
Using data obtained before the initiation of the study
question
6. Nurses need to become familiar with the elements of a research publication. A brief explanation of the type of measurement to be used is found in which section of a study? 1. Results 2. Methods 3. Conclusion 4. Introduction
answer
Methods
question
7. After identifying the problem, the next step in the research process is to: 1. Select the population 2. Review the literature 3. Obtain approval to conduct the study 4. Identify the instrument to use for data analysis
answer
Review the literature
question
8. A sample of orthopedic clients varies greatly in their requests for postsurgical analgesics. Which type of nursing research would best examine a prospective group of clients in determining what factors affect their alterations in comfort? 1. Historical research 2. Evaluation research 3. Correlational research 4. Experimental research
answer
Correlational research
question
9. Which of the following research topics best lends itself to the experimental research process method? 1. The effects of therapeutic touch on a geriatric client diagnosed with Alzheimer's disease 2. Prioritizing three nursing diagnoses for a newly admitted client with diabetes mellitus 3. Employing humor as an intervention with clients who are recovering from orthopedic surgery 4. Determining the blood pressure patterns of a client who recently experienced a cerebrovascular accident (i.e., stroke)
answer
Employing humor as an intervention with clients who are recovering from orthopedic surgery
question
10. The nurse is looking at different strategies for learning and incorporating new information into practice. A strategy that uses problem-solving is demonstrated by: 1. Repeatedly practicing vital signs until competence is achieved 2. Seeking information from the nurse manager on the client's status 3. Reviewing Maslow's hierarchy either in a textbook or on the internet 4. Trying different types of colostomy dressings for maximum therapeutic effect
answer
Trying different types of colostomy dressings for maximum therapeutic effect
question
11. A nurse researcher has completed a study involving the use of intravenous analgesics for postsurgical discomfort. The description of the 16 clients used for the study would best be written in which part of the research report? 1. Results section 2. Methods section 3. Discussion section 4. Introduction section
answer
Methods section
question
14. A nurse routinely uses therapeutic touch when caring for postoperative clients with incisional pain. Occasionally a client will show reluctance when the intervention is offered. The nurse's best response in such a situation is to: 1. Research for alternative interventions that will be better received by the client 2. Suggest that the client allow the intervention just once before making a final decision 3. Respect the client's wishes and rely on pain medication to help with managing the pain 4. Inform the client that the intervention has been found to be effective during several research projects
answer
Research for alternative interventions that will be better received by the client
question
2. An industrial nurse is planning to give an informative talk on hypertension to employees in honor of "heart month." He plans to teach individuals how to take their blood pressure measurements. Which information is important for him to ask the planning committee before this presentation? 1. Ages of all employees involved 2. Names of employees who are married 3. Number of employees with high blood pressure 4. Type of room available and number of participants
answer
Type of room available and number of participants
question
4. There are a variety of teaching methodologies fro a nurse to choose from to use with clients. For a toddler, the nurse should use: 1. Role-playing 2. Problem-solving 3. Independent learning 4. Simple explanations and pictures
answer
Simple explanations and pictures
question
5. The nurse has important information to share with a parent who has brought his child to the emergency department. The nurse discovers that the parent, who appears very anxious, has just learned his son will require surgery. The most effective teaching approach in this situation is: 1. Telling 2. Trusting 3. Participating 4. Group teaching
answer
Telling
question
6. A client, after being taught of the clinical manifestations of inflammation to enable early detection of a complication of a surgical wound states, "I will look at the wound four times a day and tell my surgeon if it looks red or swollen." Her statement is an example of: 1. Attitudes 2. Application 3. Analysis 4. Evaluation
answer
Application
question
8. There are many factors are assessed before teaching the client to learn insulin injection sites, but the most important factor for the nurse to assess first is the: 1. Previous knowledge level of the client 2. Willingness of the client to want to learn the injection sites 3. Financial resources available to the client for the equipment 4. Intelligence and developmental level of the individual client
answer
Willingness of the client to want to learn the injection sites
question
10. Clients give various responses to teaching sessions. For the nurse, an example of an evaluation of a psychomotor skill is: 1. Client states side effects of a medication 2. Client responds appropriately to eye contact 3. Client independently plans an exercise program 4. Client demonstrates the proper use of a walking cane
answer
Client demonstrates the proper use of a walking cane
question
11. Different topics are presented in the information sessions that are held in the outpatient clinic. In planning for a session on health maintenance/illness prevention, the nurse should select a topic on: 1. Use of assistive devices, such as canes 2. Self-help devices for post-CVA clients 3. Stress management techniques for working parents 4. Environmental alterations for clients in wheelchairs
answer
Stress management techniques for working parents
question
12. The nurse is evaluating the responses of clients to teaching sessions. An example of an evaluation of a client's attainment of a cognitive skill is: 1. Client explains that the medication should be taken with meals 2. Client looks at the surgical incision without requiring prompting 3. Client uses crutches appropriately to move both up and down stairs 4. Client independently capable of dressing self after eating breakfast
answer
Client explains that the medication should be taken with meals
question
13. The nurse evaluates which of the following statements as an indication that the client is not ready to learn at this time? 1. "I need to understand more about the reason for the colostomy." 2. "I will find out more about that when the support group meets." 3. "There's no sense in showing me that now. I'm too sick right now." 4. "Please be sure to tell me if I am completing all the steps correctly."
answer
"There's no sense in showing me that now. I'm too sick right now."
question
15. The nurse has completed an assessment on the client and identified the following nursing diagnoses. Which one of the following nursing diagnoses indicates a need to postpone teaching that was planned? 1. Activity intolerance related to pain 2. Ineffective management of treatment regimen 3. Noncompliance with prescribed exercise plan 4. Knowledge deficit regarding impending surgery
answer
Activity intolerance related to pain
question
16. There are a variety of teaching methodologies that may be utilized to meet the client's needs. Which teaching method is best applied to a cognitive learning need? 1. Modeling of behavior 2. Discussion of feelings 3. Computer-assisted instruction 4. Demonstration of a procedure
answer
Computer-assisted instruction
question
17. For a functionally illiterate client, the nurse particularly focuses on: 1. Using intricate analogies and examples 2. Avoiding lengthy return demonstrations 3. Incorporating familiar nonmedical terminology 4. Providing longer learning sessions with the client
answer
Incorporating familiar nonmedical terminology
question
18. In preparing a teaching plan for adult clients in a cancer support group, the nurse incorporates evidence-based information. The nurse recognizes that evidence obtained about adult learners has identified that this group prefers: 1. Computer-assisted instruction 2. Traditional classroom settings 3. Long sessions with plenty of technical information 4. Interesting personal communication techniques
answer
Interesting personal communication techniques
question
19. While teaching the client about management of his heart disease, a nurse might use a strategy that is implemented to promote learning in the affective domain such as: 1. Asking the client what he believes he needs to know about the diagnosis 2. Providing brochures both on current exercises and on nutrition guidelines 3. Encouraging the client to personally discuss his feelings about his health status 4. Having the client return-demonstrate self-measurement of his own blood pressure
answer
Encouraging the client to personally discuss his feelings about his health status
question
20. The nurse is preparing to present a teaching session on skin protection for a group of older adults at a senior center. A principle that has been found to be most effective in teaching older adults is: 1. Moving the group along at a predetermined pace 2. Providing information in longer teaching sessions 3. Speaking very slowly and in a louder tone of voice 4. Beginning and ending each session with important information
answer
Beginning and ending each session with important information
question
21. The nurse is preparing the discharge teaching materials on newly prescribed drugs to a client diagnosed to be in the early stage of Alzheimer's disease. The nurse best deals with the client's cognitive deficits by: 1. Providing written material to supplement the discussion 2. Arranging for family to be present during the discussion 3. Presenting the material in two short but focused sessions 4. Requiring the client to restate the information in her own words
answer
Arranging for family to be present during the discussion
question
22. The nurse recognizes that the client's teaching plan is most directly driven by: 1. The client's identified learning needs 2. The complexity of the client's health needs 3. The client's readiness and motivation to learn 4. The presence of cultural or physical barriers
answer
The client's identified learning needs
question
23. The nurse recognizes that the primary goal of a client's teaching plan is to: 1. Facilitate a knowledge-based client decision-making process 2. Provide information that brings about informed client consent 3. Enhance the client's sense of personal control regarding his or her health care 4. Therapeutically affect the client's health, wellness, and independence
answer
Therapeutically affect the client's health, wellness, and independence
question
25. Which of the following teaching topics is an example of restoration of health? 1. Glucose monitoring at home 2. Living with rheumatoid arthritis 3. Stress management's impact on depression 4. What to expect after hip replacement surgery
answer
What to expect after hip replacement surgery
question
26. Which of the following actions is the primary nursing responsibility regarding client education? 1. Providing accurate, current, relevant information 2. Answering the client's questions regarding health-related issues 3. Assessing the individual client's readiness and motivation to learn 4. Identifying areas where clients are in need of educational information
answer
Providing accurate, current, relevant information
question
27. When a client newly diagnosed with type 2 diabetes mellitus assumes responsibility for checking her blood glucose level four times a day, this is an example of: 1. Cognitive learning 2. Affective learning 3. Impaired learning 4. Psychomotor learning
answer
Psychomotor learning
question
28. When a client newly diagnosed with type 2 diabetes mellitus selects a lunch menu that correlates with the number of carbohydrates he is allowed for that meal, this is an example of: 1. Cognitive learning 2. Affective learning 3. Impaired learning 4. Psychomotor learning
answer
Cognitive learning
question
29. Which of the following statement best reflects the nurse's appropriate attention to a client's need for self-efficacy? 1. "What can I do to help you lose the weight?" 2. "Are you really ready to start a regular exercise regimen?" 3. "After you watch me demonstrate this inhaler, you will have no problems using it at all." 4. "Come on; with all the self-help products out there, you will be able to stop smoking."
answer
"After you watch me demonstrate this inhaler, you will have no problems using it at all."
question
30. A client has been recently told that the primary cancer has metastasized, and the cancer is considered terminal. When the nurse offers to discuss palliative care options, the client replies, "I'm going to have the reports reevaluated by another doctor; I feel fine and I think a mistake has been made." The nurse recognizes this response as: 1. Anger 2. Disbelief 3. Bargaining 4. Acceptance
answer
Disbelief
question
31. A client has been recently told that the primary cancer has metastasized and the cancer is considered terminal. When the nurse offers to discuss palliative care options the client replies, "I can't understand why you all want to upset me by bringing the topic up. Now please just leave me alone." The nurse recognizes this response as: 1. Anger 2. Disbelief 3. Bargaining 4. Acceptance
answer
Anger
question
Which of the following is a current trend in families of family living? 1. People marrying earlier 2. Reduction in the divorce rate 3. People having more children 4. More people choosing to live alone
answer
More people choosing to live alone
question
**Of the following trends, which one represents the greatest current health care challenge to nurses? 1. Homelessness 2. Single parent families 3. Alternative relationship patterns 4. "Sandwiched" or middle geration
answer
Homelessness
question
When working with families, the nurse may view the family as context or client. Which one of the following examples demonstrates the view of the family as context? 1. The family's ability to support the client's dietary and recreational needs 2. The client's ability to understand and manage his own personal dietary needs 3. The family's demands on the client that are based on the client's role performance 4. The adjustment of both the client and the family to changes in diet and exercise
answer
The client's ability to understand and manage his own personal dietary needs
question
**What would a nurse expect to find in an assessment of a healthy family? 1. Change is viewed as detrimental to the family 2. There is a passive response to most stressors 3. The structure is flexible enough to adapt to crises 4. Minimum influence is being exerted on the environment
answer
The structure is flexible enough to adapt to crises
question
Initially, the nurse should begin by doing what in completing a client's family assessment? 1. Collecting health data from all the family members 2. Testing the family's ability to cope with normal stressors 3. Evaluating the family's interpersonal communication patterns 4. Determining the client's definition of familiar structure and attitudes
answer
Determining the client's definition of familiar structure and attitudes
question
**Post discharge, the client is returning to their home environment. In assisting the client with that, specifically in implementing family-centered care, the nurse: 1. Provides personal beliefs regarding problem-solving 2. Assists the family members to assume dependent roles 3. Works with the client to accept responsibility for role in discourse 4. Offers both client and family information about necessary self-care abilities
answer
Offers both client and family information about necessary self-care abilities
question
**A client is unable to independently perform colostomy care due to arthritis. The nurse should first: 1. Offer to assist the client to learn to manage the care 2. Arrange for home care services to care for the colostomy 3. Inquire as to family members who may be able to assist with the care 4. Suggest that the client attend a colostomy self-help support group
answer
Inquire as to family members who may be able to assist with the care
question
**The optimum goal of effective communication within the family, according to the nurse observing the family members and their interaction, is: 1. Problem solving and psychological support 2. Role development of individual members 3. Socialization among individual members 4. Better financial conditions for the family
answer
Problem solving and psychological support
question
Which of the following is a gerontological principle related to families? 1. Later-life families need not work on developmental tasks. 2. The caregivers are often not members of the client's family. 3. Role reversal is usually expected and well accepted by the older client. 4. Support systems are likely to be different than those of younger age-groups.
answer
Support systems are likely to be different than those of younger age-groups.
question
**In assessing the roles and power structure of a client's nuclear family, the nurse should specifically ask the client: 1. "Who decides where to go on vacation?" 2. "What type of health care insurance do you have?" 3. "How many family members currently live in your home?" 4. "What types of social activities do you and your family enjoy?"
answer
"Who decides where to go on vacation?"
question
**Needing assistance with daily living activities, an older adult with two grown children is being discharged home. Although both children live nearby, the daughter is expressing concern about handling her parent's physical needs. The nurse's initial response is to: 1. Work with the family on delegating responsibility 2. Suggest short-term nursing home placement to the client 3. Arrange for the client to remain hospitalized in the medical center 4. Make decisions for the family on how to manage the care at home
answer
Work with the family on delegating responsibility
question
The nurse suspects that there is physical abuse present after visiting the client in the home. In recognition of the pattern of family violence, the nurse knows that: 1. Child abuse is declining in frequency 2. Spouses are the most frequent abusers 3. Mental illness is a major cause of abuse 4. Abuse is primarily seen in lower income families
answer
Spouses are the most frequent abusers
question
**The primary goal of family-centered nursing is to: 1. Promote the wellness of the family and its members 2. Implement appropriate care for the family and its members 3. Provide support and care for the family and its individual members 4. Identify physical and emotional problems affecting the family as a unit
answer
Provide support and care for the family and its individual members
question
**A nurse who is sensitive to the care of families recognizes that the term "family" is primarily defined: 1. As individuals legally bound to the client 2. As people with biological connections to the client 3. In terms generally accepted by the majority of clients 4. By the client as individuals important to the client
answer
By the client as individuals important to the client
question
The nurse is preparing a new mother for discharge when the woman shares that she is "worried about going back to work and its effects on my infant." The most therapeutic response by the nurse is: 1. "Do you want to go back to work?" 2. "Just be sure you have an excellent baby sitter." 3. "There is no proof that working will harm your baby." 4. "Can your husband share in the child care reponsibilities?"
answer
"There is no proof that working will harm your baby."
question
The gratest risk to a child of adolescent parents comes from the: 1. Increased family stressors resulting in domestic violence 2. Lack of appropriate parenting resources and role models 3. Statically high potential for physical and emotional abuse 4. Parents inability to provide health care and economc support
answer
Lack of appropriate parenting resources and role models
question
**Which of the following nursing statements has the greatest therapeutic value when counseling a "sandwich generation" client caring for a chronically ill parent? 1. "I can help you in finding assistance with the in-home care." 2. "What is the most stressful aspect of caring for your parent?" 3. "I'm sure your children love having grandmother in the house." 4. "What do you do for relaxation now that your mom lives with you?"
answer
"What do you do for relaxation now that your mom lives with you?"
question
**The mother of a child receiving immunizations at a health clinic shares with the nurse that she and the child have not eaten today. Which of the following nursing interventions is best directed at impacting the immediate problem while being sensitive to the mother's sense of self-worth? 1. Notifying family services of the problem 2. Taking both mother and child to the cafeteria 3. Informing the mother that she is eligible for food stamps 4. Providing her with contacts at the neighborhood food bank
answer
Providing her with contacts at the neighborhood food bank
question
**The nurse recognizes that the presence of an alcohol-abusing parent places a child at greatest risk for: 1. Homelessness 2. School truancy 3. Family violence 4. Accident-related injuries
answer
Family violence
question
**The most important impact that truthful, timely communication between the nurse and the family of a critically ill client has is on the family's ability to: 1. Trust the nurse 2. Adjust to "bad news" 3. Be confident of the care the client is receiving 4. Make appropriate choices regarding client treatment
answer
Trust the nurse
question
**When caring for a terminally ill client, the nurse must also assess the family, because the primary benefit will be: 1. Effective use of time and resources in the end-of-life care of the client 2. Appropriate attention to the cultrual beliefs and expectations of the family 3. Added information regarding the care needs and preferences of the client 4. The ability to respond effectively to the familly unit during the dying process
answer
The ability to respond effectively to the familly unit during the dying process
question
When attempting to meet the needs of the family, the nurse recognizes the central concept of the theory of family developmental stages is that: 1. Over time all families progress through developmental stages 2. Needs differ as the family progresses through the various stages 3. While each family is unique, they all tend to progress through similar stages 4. The family will progress only when all the challenges of a particular stage are met
answer
While each family is unique, they all tend to progress through similar stages
question
**The nurse can primarily affect the effectiveness of a family's ability to cope with stress by encouraging: 1. Flexible roles 2. Distinct task assignment 3. Individual independence 4. Variable parenting models
answer
Flexible roles
question
The nurse is providing diabetic diet teaching to a Hispanic man and his wife. When the nurse is discussing foods that are acceptable, the wife continues to interrupt with statements like, "Oh, he doesn't eat that," or, "All he eats is rice and beans." What should the nurse do? A. Ask the wife to leave so he/she can focus on teaching the patient. b.Explain how "rice and beans" are not acceptable foods on a diabetic diet. c.Provide a diet plan with only food alternatives selected by the patient. d.Refer the patient and his wife to a dietitian familiar with Spanish food choices.
answer
Ans: D. Refer the patient and his wife to a dietitian familiar with Spanish food choices. The nurse should refer the patient to speak with a dietitian who is familiar with cultural food choices. If possible, he/she should develop a diet plan that includes the patient's cultural diet preferences and can provide culturally sensitive teaching brochures that describe healthy food choices. Rice and beans may be acceptable alternatives in a balanced diet. The nurse should include people in the family who help shop for and prepare food in the home, along with the wife.
question
The nurse is caring for a patient who requires a complex dressing change. While in the patient's room, the nurse decides to change the dressing. What does the nurse do just before changing the dressing? a.Assesses the patient's readiness for the procedure b.Gathers and organizes needed supplies c.Decides on goals and outcomes for the patient d.Calls for assistance from another nursing staff member
answer
a. Assesses the patient's readiness for the procedure The nurse needs to assess the patient's readiness and willingness for any procedure before intervening. After determining the patient's readiness for the dressing change, the nurse gathers needed supplies. The nurse establishes goals and outcomes before intervening. Before entering the patient's room, the nurse needs to ask another staff member to help if necessary.
question
A patient visiting with family members in the waiting area tells the nurse that his stomach is not feeling good. Before intervening, what should the nurse do? a.Ask the patient to return to his room so the nurse can inspect his abdomen. b.Request that the family leave, so the patient can rest. c.Ask the patient when his last bowel movement was and to lie down on the sofa. d.Tell the patient that his dinner tray will be ready in 15 minutes.
answer
A. Ask the patient to return to his room so the nurse can inspect his abdomen. Assessment is the first step in the nursing process and needs to be completed before the nurse can intervene. In this case, the environment needs to be conducive to completing a thorough assessment. The patient needs to return to the room for an abdominal assessment for privacy and comfort. The family can remain in the waiting area while the nurse assists the patient back to the room. Beginning the assessment in the waiting area in the presence of family and other visitors does not promote privacy and patient comfort. Telling the patient that his dinner tray is almost ready is making an assumption that the abdominal discomfort is due to not eating. The nurse needs to perform an assessment first.
question
A newly admitted patient who is morbidly obese asks the nurse to assist her to the bathroom for the first time. What should the nurse do first? a.Ask for at least two other assistive personnel to come to the room. b.Medicate the patient to alleviate discomfort while ambulating. c.Offer the patient a walker. d.Review the patient's activity orders.
answer
ANS: D Before intervening, the nurse must check the patient's orders. For example, if the patient is on bed rest, the nurse will need to explain the use of a bedpan rather than helping the patient get out of bed to go to the bathroom. Interventions sometimes will be determined by orders and availability of resources. Asking for assistive personnel is appropriate after making sure the patient can get out of bed. If the patient is obese, the nurse will likely need assistance in getting the patient to the bathroom. Medicating the patient before checking the orders is not advised in this situation. Before medicating for pain, the nurse needs to perform a pain assessment. Offering the patient a walker is a premature intervention until the orders are verified.
question
Which of these interventions, to be included in the plan of care, is appropriate for the patient outcome that states, "The patient will verbalize a pain level at 3 or below on a 0 to 10 scale throughout this shift."? a.Medicate the patient immediately after all procedures. b.Discuss only nonpharmacological methods of pain relief. c.Teach the patient about side effects of pain medications. d.Medicate the patient based on previous shift assessment findings.
answer
ANS: C The nurse needs to include teaching as an appropriate nursing intervention. Medicating the patient after procedures is not a helpful method of pain control. Patients need to be assessed for sign and symptoms of discomfort before and after procedures. The nurse discusses all options for pain relief, not just nonpharmacological methods. Patients' needs can change from minute to minute, so basing an intervention on a previous shift assessment is incorrect.
question
The patient has presented to the ambulatory surgery center to have a colonoscopy. The patient is scheduled to receive moderate sedation (conscious sedation) during the procedure. Moderate sedation is used routinely for procedures that require a.Performance on an outpatient basis. b.A depressed level of consciousness. c.Loss of sensation in an area of the body. d.The patient to be immobile.
answer
ANS: B Moderate sedation (conscious sedation) is used routinely for procedures that do not require complete anesthesia, but rather a depressed level of consciousness. Not all patients who are treated on an outpatient basis receive moderate sedation. Regional anesthesia such as local anesthesia provides loss of sensation in an area of the body. General anesthesia is used for patients who need to be immobile and to not remember the surgical procedure.
question
The nurse is caring for a patient in the postanesthesia care unit who has undergone a left total knee arthroplasty. The anesthesia provider has indicated that the patient received a left femoral peripheral nerve block. Which assessment would be an expected finding for a patient with this type of regional block? a.Decreased pulse at the left posterior tibia b.Left toes cool to touch and slightly cyanotic c.Sensation decreased in the left leg d.Patient report of pain in the left foot
answer
ANS: C Induction of regional anesthesia results in loss of sensation in an area of the body. The peripheral nerve block influences the portion of sensory pathways that are anesthetized in the targeted area of the body. Decreased pulse, toes cool to touch, and cyanosis are indications of decreased blood flow and are not expected findings. Reports of pain the in the left foot may indicate that the block is not working or is subsiding and is not an expected finding in the immediate postoperative period.
question
The nurse is completing a medication history for the surgical patient in preadmission testing. Which of the following medications should the nurse instruct the patient to hold in preparation for surgery? a.Ibuprofen b.Acetaminophen c.Vitamin C d.Miconazole
answer
ANS: A Nonsteroidal anti-inflammatory drugs (NSAIDs) such as ibuprofen inhibit platelet aggregation and prolong bleeding time, increasing susceptibility to postoperative bleeding. Acetaminophen is a pain reliever that has no special implications for surgery. Vitamin C actually assists in wound healing and has no special implications for surgery. Miconazole is an antifungal and has no special implications for surgery.
question
The nurse is caring for a potential surgical patient in the preadmission testing unit. The medication history indicates that the patient is currently taking warfarin (Coumadin). Which of the following actions should the nurse take? a.Consult with the physician regarding a radiological examination of the chest. b.Consult with the physician regarding an international normalized ratio (INR). c.Consult with the physician regarding blood urea nitrogen (BUN). d.Consult with the physician regarding a complete blood count (CBC).
answer
ANS: B Warfarin is an anticoagulant that is utilized for different maladies, but its action is to increase the time it takes for the blood to clot. This action can put the surgical patient at risk for bleeding tendencies. Typically, if at all possible, this medication is held several days before a surgical procedure to decrease this risk. INR, PT (prothrombin time), APTT (activated partial thromboplastin time), and platelet counts reveal the clotting ability of the blood. Chest x-ray, BUN, and CBC are diagnostic screening tools for surgery but are not specific to warfarin.
question
The nurse is caring for a preoperative patient. The nurse teaches the principles and demonstrates leg exercises for the patient. The patient is unable to perform leg exercises correctly. What is the nurse's best next step? a.Assess for the presence of anxiety, pain, or fatigue. b.Ask the patient why he does not want to do the exercises. c.Encourage the patient to practice at a later date. d.Assess the educational methods used to educate the patient.
answer
ANS: A If the patient is unable to perform leg exercises after sound educational principles and demonstration are provided, the nurse should look for circumstances that may be impacting the patient's ability to learn. In this case, the patient can be anticipating the upcoming surgery and may be experiencing anxiety. The patient may also be in pain or may be fatigued; both of these can affect the ability to learn. Assessment of educational methods may be needed, but in this case, sound principles and demonstration are being utilized. Asking anyone why can cause defensiveness and may not help in attaining the answer. In this case, the patient really may want to participate and may not know why he is unable to learn. The nurse is aware that the patient is unable to do the exercises. Moving forward without ascertaining that learning has occurred will not help the patient in meeting goals.
question
Which nursing assessment would indicate that the patient is performing diaphragmatic breathing correctly? a.Hands placed on border of rib cage with fingers extended will touch as chest wall contracts. b.Hands placed on chest wall with fingers extended will separate as chest wall contracts. c.The patient will feel upward movement of the diaphragm during inspiration. d.The patient will feel downward movement of the diaphragm during expiration.
answer
ANS: A Positioning the hands along the borders of the rib cage allows the patient to feel movement of the chest and abdomen as the diaphragm descends and the lungs expand. As the patient takes a deep breath and slowly exhales, the middle fingers will touch while the chest wall contracts. The fingers will separate as the chest wall expands. The patient will feel normal downward movement of the diaphragm during inspiration and normal upward movement during expiration.
question
The nurse has administered an anxiolytic as a preoperative medication to the patient going to surgery. Which of the following is the best next step? a.Waste any unused medication according to policy. b.Notify the operating suite that the medication has been given. c. Instruct the patient to call for help to go to the restroom. d.Ask the patient to sign the consent for surgery.
answer
ANS: C Once a medication has been administered, instruct the patient to call for help when getting out of bed to prevent falls. For patient safety, explain the purpose of a preoperative medication and its effects. Reinforce to the patient to stay in the bed or on the stretcher. Raise the side rails and keep the bed or stretcher in the low position. Place the call light within easy reach of the patient. Notifying the operating suite that the medication has been given may be part of a facilities procedure but is not the best next step. It is important to have the patient sign consents before the patient has received medication that may make him/her drowsy. Wasting unused medication according to policy is important but is not the best next step.
question
The ambulatory surgical nurse calls to check on the patient at home the morning after surgery. The patient is reporting continued nausea and vomiting. Which of the following discharge education points should be reviewed with the patient? a.Instruct the patient to take deep breaths. b.Instruct the patient to drink ginger ale and eat crackers. c.Instruct and attempt to connect the patient with the physician. d.Instruct the patient to go to the emergency department.
answer
ANS: C Postoperative nausea and vomiting sometimes occur once the patient is at home even if symptoms were not present in the surgery center. Options for therapy include medications. Instructing the patient to call the physician and connecting the patient with the physician can help the patient to obtain relief. Taking deep breaths, drinking ginger ale, and eating crackers are interventions that may be helpful, but this patient needs additional help. Instructing the patient to go to the emergency department is an option with continued nausea and vomiting.
question
The nurse is caring for a patient who is experiencing a full-thickness repair. The nurse would expect to see which of the following in this type of repair? a.Eschar b.Slough c.Granulation d.Purulent drainage
answer
ANS: C Granulation tissue is red, moist tissue composed of new blood vessels, the presence of which indicates progression toward healing. Soft yellow or white tissue is characteristic of slough—a substance that needs to be removed for the wound to heal. Black or brown necrotic tissue is called eschar, which also needs to be removed for a wound to heal. Purulent drainage is indicative of an infection and will need to be resolved for the wound to heal.
question
The nurse is caring for a patient in the burn unit. The nurse recalls that this type of wound heals by a.Tertiary intention. b.Secondary intention. c.Partial-thickness repair. d.Primary intention.
answer
ANS: B A wound involving loss of tissue such as a burn or a pressure ulcer or laceration heals by secondary intention. The wound is left open until it becomes filled with scar tissue. It takes longer for a wound to heal by secondary intention; thus the chance of infection is greater. A clean surgical incision is an example of a wound with little loss of tissue that heals by primary intention. The skin edges are approximated or closed, and the risk for infection is low. Partial- thickness repair are done on partial-thickness wounds that are shallow, involving loss of the epidermis and maybe partial loss of the dermis. These wounds heal by regeneration because the epidermis regenerates. Tertiary intention is seen when a wound is left open for several days, and then the wound edges are approximated. Wound closure is delayed until the risk of infection is resolved.
question
Which nursing observation would indicate that a wound healed by secondary intention? a.Minimal scar tissue b.Minimal loss of tissue function c.Permanent dark redness at site d.Scarring can be severe.
answer
ANS: D A wound healing by secondary intention takes longer than one healing by primary intention. The wound is left open until it becomes filled with scar tissue. If the scarring is severe, permanent loss of function often occurs. Wounds that heal by primary intention heal quickly with minimal scarring. Scar tissue contains few pigmented cells and has a lighter color than normal skin.
question
The nurse is caring for a patient who has experienced a total hysterectomy. Which nursing observation would indicate that the patient was experiencing a complication of wound healing? a.The incision site has started to itch. b.The incision site is approximated. c.The patient has pain at the incision site. d.The incision has a mass, bluish in color.
answer
ANS: D A hematoma is a localized collection of blood underneath the tissues. It appears as swelling, change in color, sensation, or warmth or a mass that often takes on a bluish discoloration. A hematoma near a major artery or vein is dangerous because it can put pressure on the vein or artery and obstruct blood flow. Itching of an incision site can be associated with clipping of hair, dressings, or possibly the healing process. Incisions should be approximated with edges together. After surgery, when nerves in the skin and tissues have been traumatized by the surgical procedure, it is expected that the patient would experience pain.
question
A patient has developed a decubitus ulcer. What laboratory data would be important to gather? a.Serum albumin b.Creatine kinase c.Vitamin E d.Potassium
answer
ANS: A Normal wound healing requires proper nutrition. Serum proteins are biochemical indicators of malnutrition, and serum albumin is probably the most frequently measured of these parameters. The best measurement of nutritional status is prealbumin because it reflects not only what the patient has ingested, but also what the body has absorbed, digested, and metabolized. Measurement of creatine kinase helps in the diagnosis of myocardial infarcts and has no known role in wound healing. Potassium is a major electrolyte that helps to regulate metabolic activities, cardiac muscle contraction, skeletal and smooth muscle contraction, and transmission and conduction of nerve impulses. Vitamin E is a fat-soluble vitamin that prevents the oxidation of unsaturated fatty acids. It is believed to reduce the risk of coronary artery disease and cancer. Vitamin E has no known role in wound healing.
question
hich of the following would be the most important piece of assessment data to gather with regard to wound healing? a.Muscular strength assessment b.Sleep assessment c.Pulse oximetry assessment d.Sensation assessment
answer
ANS: C Oxygen fuels the cellular functions essential to the healing process; the ability to perfuse tissues with adequate amounts of oxygenated blood is critical in wound healing. Blood flow through the pulmonary capillaries provides red blood cells for oxygen attachment. Oxygen diffuses from the alveoli into the pulmonary blood; most of the oxygen attaches to hemoglobin molecules within the red blood cells. Red blood cells carry oxygenated hemoglobin molecules through the left side of the heart and out to the peripheral capillaries, where the oxygen detaches, depending on the needs of the tissues. Pulse oximetry measures the oxygen saturation of blood. Assessment of muscular strength and sensation, although useful for fitness and mobility testing, does not provide any data with regard to wound healing. Sleep, although important for rest and for integration of learning and restoration of cognitive function, does not provide any data with regard to wound healing.
question
The nurse is caring for a patient with a healing stage III pressure ulcer. Upon entering the room, the nurse notices an odor and observes a purulent discharge, along with increased redness at the wound site. What is the next best step for the nurse? a.Complete the head-to-toe assessment, and include current treatment, vital signs, and laboratory results. b.Notify the charge nurse about the change in status and the potential for infection. c.Notify the physician by utilizing Situation, Background, Assessment, and Recommendation (SBAR). d.Notify the wound care nurse about the change in status and the potential for infection.
answer
ANS: A The patient is showing signs and symptoms associated with infection in the wound. It is serious and needs treatment but is not a life-threatening emergency, where care is needed immediately or the patient will suffer long-term consequences. The nurse should complete the assessment; gather all data such as current treatment modalities, medications, vital signs including temperature, and laboratory results such as the most recent complete blood count or white cell count. The nurse can then notify the physician and receive treatment orders for the patient. It is important to notify the charge nurse and consult the wound nurse on the patient's status and on any new orders.
question
The nurse is completing an assessment on an individual who has a stage IV pressure ulcer. The wound is odorous, and a drain is currently in place. The nurse determines that the patient is experiencing issues with self-concept when the patient states which of the following? a."I think I will be ready to go home early next week." b."I am so weak and tired, I want to feel better." c."I am ready for my bath and linen change as soon as possible." d."I am hoping there will be something good for dinner tonight."
answer
ANS: C The patient's psychological response to any wound is part of the nurse's assessment. Body image changes can influence self-concept. Factors that affect the patient's perception of the wound include the presence of scars, drains, odor from drainage, and temporary or permanent prosthetic devices. The wound is odorous, and a drain is in place. The patient who is asking for a bath and change in linens gives you a clue that he or she may be concerned about the smell in the room. The patient stating that he or she wants to feel better, talking about going home, and caring about what is for dinner could be interpreted as positive statements that indicate progress along the health journey.
question
The nurse is caring for a patient with a stage III pressure ulcer. The nurse has assigned a nursing diagnosis of Risk for infection. Which intervention would be most important for this patient? a.Teach the family how to manage the odor associated with the wound. b.Discuss with the family how to prepare for care of the patient in the home. c.Encourage thorough handwashing of all individuals caring for the patient. d.Encourage increased quantities of carbohydrates and fats.
answer
ANS: C The number one way to decrease the risk of infection by breaking the chain of infection is to wash hands. Encouraging fluid and food intake helps with overall wellness and wound healing, especially protein, but an increase in carbohydrates and fats does not relate to the risk of infection. If the patient will be discharged before the wound is healed, the family will certainly need education on how to care for the patient. Teaching the family how to manage the odor associated with a wound is certainly important, but these interventions do not directly relate to the risk of infection and breaking the chain of the infectious process.
question
The nurse is caring for a patient with a stage II pressure ulcer and has assigned a nursing diagnosis of Risk for infection. The patient is unconscious and bedridden. The nurse is completing the plan of care and is writing goals for the patient. What is the best goal for this patient? a.The patient's family will demonstrate specific care of the wound site. b.The patient will state what to look for with regard to an infection. c.The patient will remain free of an increase in temperature and of odorous or purulent drainage from the wound. d.The patient's family members will wash their hands when visiting the patient.
answer
ANS: C Because the patient has an open wound and the skin is no longer intact to protect the tissue, the patient is at increased risk for infection. The nurse will be assessing the patient for signs and symptoms of infection, including an increase in temperature, an increase in white count, and odorous and purulent drainage from the wound. The patient is unconscious and is unable to communicate the signs and symptoms of infection; also, this is an intervention, not a goal for this diagnosis. It is important for the patient's family to be able to demonstrate how to care for the wound and wash their hands, but these statements are interventions, not goals or outcomes for this nursing diagnosis.
question
The nurse is caring for a postpartum patient. The patient has an episiotomy after experiencing birth. The physician has ordered heat to treat this condition, and the nurse is providing this treatment. This patient is at risk for a.Infection. b.Impaired skin integrity. c.Trauma. d.Imbalanced nutrition.
answer
ANS: C Heat causes vasodilatation and is used to improve blood flow to an injured body part. The application of heat incorrectly when the treatment is too hot, or is applied too long or to the wrong place, can result in a burn for the patient and risk for additional trauma. The skin already has impaired integrity owing to the surgical procedure, and because of this has been at risk for infection since the surgical procedure was performed. This patient is of childbearing age and has had a child. Additional needs for nutrition are present during pregnancy and breastfeeding, but this is an established nursing diagnosis. Data are insufficient to support the nursing diagnosis of Imbalanced nutrition.
question
Although isometric contractions do not result in muscle shortening, the nurse understands that isometric contractions a.Result in decreased energy expenditure. b.Are always desirable regardless of patient condition. c.Are necessary for the active movement of muscles. d.Result in increased energy expenditure.
answer
ANS: D Although isometric contractions do not result in muscle shortening, energy expenditure increases. It is important to understand the energy expenditure associated with isometric exercises because they are sometimes contraindicated in certain illnesses. Isometric contractions increase muscle tension but not active movement of the muscle.
question
Of the following nursing goals, which is the most appropriate for a patient who has had a total hip replacement? a.The patient will walk 1000 feet using her walker by the time of discharge. b.The patient will ambulate by the time of discharge. c.The patient will ambulate briskly on the treadmill by the time of discharge. d.The nurse will assist the patient to ambulate in the hall.
answer
ANS: A "The patient will walk 1000 feet using her walker by the time of discharge" is individualized, realistic, and measurable. "The patient will ambulate by the time of discharge" is not measurable because it does not specify the distance. Even though we can see that the patient will ambulate, this does not quantify how far. "Ambulating briskly on a treadmill" is not realistic for this patient. The last option focuses on the nurse, not the patient, and is not measurable.
question
he nurse is caring for a patient who has had a stroke causing total paralysis of the right side. To help maintain joint function and to prevent contractures, passive ROM will be initiated. When should therapy begin? a.After the acute phase of the disease has passed b.As soon as the ability to move is lost c.Once the patient enters the rehab unit d.No ROM is needed.
answer
ANS: B Passive ROM exercises should begin as soon as the patient's ability to move the extremity or joint is lost. The nurse should not wait for the acute phase to end. It may be some time before the patient enters the rehab unit, and contractures could form by then. ROM is certainly needed in this patient.
question
The nurse is admitting a patient who has been diagnosed as having had a stroke. The physician writes orders for "ROM as needed." The nurse understands that a.The nurse will have to move all the patient's extremities. b.The patient is unable to move his extremities. c.Further assessment of the patient is needed. d.The patient needs to restrict his mobility as much as possible.
answer
ANS: C Further assessment of the patient is needed. Some patients are able to move some joints actively, whereas the nurse passively moves others. With a weak patient, the nurse may have to support an extremity while the patient performs the movement. In general, exercises need to be as active as health and mobility allow.
question
A patient expresses concerns over having black stool. The fecal occult test is negative. Which response by the nurse is most appropriate? a."This is probably a false negative; we should rerun the test." b."Do you take iron supplements?" c."You should schedule a colonoscopy as soon as possible." d."Sometimes severe stress can alter stool color."
answer
ANS: B Certain medications and supplements, such as iron, can alter the color of stool. The fecal occult test takes three separate samples over a period of time and is a fairly reliable test. A colonoscopy is health prevention screening that should be done every 5 to 10 years; it is not the nurse's initial priority. Stress alters GI motility and stool consistency, not color.
question
nurse is pouching an ostomy on a patient with an ileostomy. Which action by the nurse is most appropriate? a.Changing the skin barrier portion of the ostomy pouch daily b.Selecting a pouch that is able to hold excess output to reduce the frequency of pouch emptying c.Thoroughly scrubbing the skin around the stoma to remove excess stool and adhesive d.Measuring the correct size for the barrier device while leaving a 1/8-inch space around the stoma
answer
ANS: B Selecting a pouch that holds a large volume of output will decrease the frequency of emptying the pouch and may ease patient anxiety about pouch overflow. The barrier device should be changed every few days unless it is leaking or is no longer effective. Peristomal skin should be gently cleansed; vigorous rubbing can cause further irritation or skin breakdown. Approximately 1/16 of an inch is present between the barrier device and the stoma. Excess space allows fecal matter to have prolonged exposure to skin, resulting in skin breakdown.
question
The nurse administers a cathartic to a patient. The nurse determines that the cathartic has had a therapeutic effect when the patient a.Has a decreased level of anxiety. b.Experiences pain relief. c.Has a bowel movement. d.Passes flatulence.
answer
ANS: C A cathartic is a laxative that stimulates a bowel movement. It would be effective if the patient experiences a bowel movement. The other options are not outcomes of administration of a cathartic.
question
A nurse is providing discharge teaching for a patient who is going home with a guaiac test. Which statement by the patient indicates the need for further education? a."If I get a positive result, I have gastrointestinal bleeding." b."I should not eat red meat before my examination." c."I should schedule to perform the examination when I am not menstruating." d."I will need to perform this test three times if I have a positive result."
answer
ANS: A A positive result does not mean GI bleeding; it could be a false positive from consuming red meat, some raw vegetables, or NSAIDs. Proper patient education is important for viable results. The patient needs to avoid certain foods to rule out a false positive. If the test is positive, the patient will need to repeat the test at least three times. Menses and hemorrhoids can also lead to false positives.
question
A nurse is caring for an older adult patient with fecal incontinence due to cathartic use. The nurse is most concerned about which complication that has the greatest risk for severe injury? a.Rectal skin breakdown b.Contamination of existing wounds c.Falls from attempts to reach the bathroom d.Cross-contamination into the upper GI tract
answer
ANS: C The nurse is most concerned about the worst injury the patient could receive, which involves falling while attempting to get to the bathroom. To reduce injury, the nurse should clear the path and reinforce use of the call light. The question is asking for the greatest risk of injury, not the most frequent occurrence or the event most likely to occur.
question
A patient has constipation and hypernatremia. The nurse prepares to administer which type of enema? a.Oil retention b.Carminative c.Saline d.Tap water
answer
ANS: D Tap water enema would draw fluid into the system and would help flush out excess sodium. Oil retention would not address sodium problems. Carminative enemas are used to provide relief from distention caused by gas. A saline enema would worsen hypernatremia.
question
A guaiac test has been ordered. The nurse knows that this is a test for a.Bright red blood. b.Dark black blood. c.Blood that contains mucus. d.Blood that cannot be seen.
answer
ANS: D Fecal occult blood tests are used to test for blood that may be present in stool that cannot be seen by the naked eye. This is usually indicative of a GI bleed. All other options are incorrect.
question
The nurse should place the patient in which position when preparing to administer an enema? a.Left Sims' position b.Fowler's c.Supine d.Semi-Fowler's
answer
ANS: A Side-lying Sims' position allows the enema solution to flow downward by gravity along the natural curve of the sigmoid colon. This helps to improve retention of the enema. Administering an enema in a sitting position may allow the curved rectal tube to scrape the rectal wall.
question
When reviewing laboratory results, the nurse should immediately notify the health care provider about which finding? a.Glomerular filtration rate of 20 mL/min b.Urine output of 80 mL/hr c.pH of 6.4 d.Protein level of 2 mg/100 mL
answer
ANS: A Normal glomerular filtration rate should be around 125 mL/min; a severe decrease in renal perfusion could indicate a life-threatening problem such as shock or dehydration. Normal urine output is 1000 to 2000 mL/day; an output of 30 mL/hr or less for 2 or more hours would be cause for concern. The normal pH of urine is between 4.6 and 8.0. Protein up to 8 mg/100 mL is acceptable; however, values in excess of this could indicate renal disease.
question
The nurse suspects that a urinary tract infection has progressed to cystitis when the patient complains of which symptom? a.Dysuria b.Flank pain c.Frequency d.Fever and chills
answer
ANS: C Cystitis is inflammation of the bladder; associated symptoms include hematuria and urgency/frequency. Dysuria is a common symptom of a lower urinary tract infection. Flank pain, fever, and chills are all signs of pyelonephritis.
question
What signs and symptoms would the nurse expect to observe in a patient with excessive white blood cells present in the urine? a.Fever and chills b.Difficulty holding in urine c.Increased blood pressure d.Abnormal blood sugar
answer
ANS: A The presence of white blood cells in urine indicates a urinary tract infection. Difficulty with urinary elimination indicates blockage or renal damage. Increased blood pressure is associated with renal disease or damage and some medications. Abnormal blood sugars would be seen in someone with ketones in the urine, as this finding indicates diabetes.
question
The nurse anticipates urinary diversion from the kidneys to a site other than the bladder for which patient? a.A 12-year-old female with severe abdominal trauma b.A 24-year-old male with severe genital warts around the urethra c.A 50-year-old male with recent prostatectomy d.A 75-year-old female with end-stage renal disease
answer
ANS: A Urinary diversion would be needed in a patient with abdominal trauma who might have injury to the urinary system. Genital warts are not needed for urinary diversion. Patients with a prostatectomy may require intermittent catheterization after the procedure. End-stage renal disease would not be affected by rerouting the flow of urine.
question
The nurse would expect to see increased ventilations if a patient exhibits a.Increased oxygen saturation. b.Decreased carbon dioxide levels. c.Decreased pH. d.Increased hemoglobin levels.
answer
ANS: C Retained CO2 creates H+ byproducts that lower pH. This sends a chemical signal to increase respiratory rate and would result in increased ventilation. All other options would cause the ventilation rate to normalize or decrease to increase carbon dioxide retention or as the result of delivery of higher levels of oxygen to tissues.
question
While performing an assessment, the nurse hears crackles in the patient's lung fields. The nurse also learns that the patient is sleeping on three pillows. What do these symptoms most likely indicate? a.Left-sided heart failure b.Right-sided heart failure c.Atrial fibrillation d.Myocardial ischemia
answer
ANS: A Left-sided heart failure results in pulmonary congestion, the signs and symptoms of which include shortness of breath, crackles, and discomfort when lying supine. Right-sided heart failure is systemic and results in peripheral edema and hepatojugular distention. Atrial fibrillation results in an irregular heart rate. Myocardial ischemia most often results in chest pain, along with shortness of breath, nausea, and fatigue.
question
he nurse caring for a patient with ischemia to the left coronary artery would expect to find a.Increased ventricular diastole. b.Increased stroke volume. c.Decreased preload. d.Decreased afterload.
answer
ANS: D The left coronary artery supplies the muscles of the left ventricle; the strength of the muscle affects the contractility of the heart. The other options are not impacted by the muscles of the left ventricle.
question
A nurse is assisting a patient with ambulation. The patient becomes short of breath and begins to complain of sharp chest pain. Which action by the nurse is the first priority? a.Call for the emergency response team to bring the defibrillator. b.Have the patient sit down in the nearest chair. c.Return the patient to the room and apply 100% oxygen. d.Ask a coworker to get the ECG machine STAT.
answer
ANS: B The patient is experiencing cardiac distress for reasons unknown. The nurse should first secure the safety of the patient and decrease the workload on the patient's heart by putting him in a resting position; this will increase cardiac output by decreasing after load. Once the patient is stable, the nurse can obtain oxygen to put on the patient. Next, the nurse can begin to monitor the patient's oxygen and cardiac status. If necessary, the emergency team may be activated to defibrillate.
question
When caring for a patient with atrial fibrillation, the nurse is most concerned with which vital sign? a.Heart rate b.Pain c.Oxygen saturation d.Blood pressure
answer
ANS: C Atrial fibrillation results in pooling of blood in the atria, forming emboli that can be pumped out to the rest of the body. The most common manifestations are stroke, myocardial infarction, and pulmonary embolus. A sudden and drastic drop in oxygenation and blood pressure can indicate both pulmonary embolus and myocardial infarction.
question
The nurse would expect a patient with right-sided heart failure to have which of the following? a.Peripheral edema b.Basilar crackles c.Chest pain d.Cyanosis
answer
ANS: A Right-sided heart failure results from inability of the right side of the heart to pump effectively, leading to a systemic backup. Peripheral edema and hepatojugular distention are signs of right-sided failure. Basilar crackles can indicate pulmonary congestion from left-sided heart failure. Cyanosis and chest pain result from inadequate tissue perfusion.
question
5-year-old who has strep throat was given aspirin for fever. The nurse knows to expect which change in the child's respiratory pattern? a.Hyperventilation to decrease serum levels of carbon dioxide b.Hypoventilation to compensate for metabolic alkalosis c.Flail chest to decrease the work of breathing d.Shallow respirations to decrease serum pH
answer
ANS: A Aspirin causes an increase in carbon dioxide; the body compensates for this by increasing ventilations to blow off excess CO2. Hypoventilation would cause the body to retain even more carbon dioxide and therefore respiratory acidosis. Flail chest occurs with trauma to the chest wall. Shallow respirations would increase serum pH.
question
A nurse is caring for a patient who suffered a myocardial infarction to the left coronary artery. Upon assessment, the nurse expects to find a.Blood in the sputum. b.Distended jugular vein. c.Peripheral edema. d.Crackles in the lungs.
answer
ANS: D The left coronary artery supplies the left ventricle of the heart; damage to the muscle in the left ventricle leads to pulmonary congestion and frothy sputum, and crackles may be heard. A distended jugular vein and peripheral edema are associated with damage to the right side of the heart. Blood in the sputum is indicative of an infection such as tuberculosis.
question
nurse caring for a patient with COPD knows that which oxygen delivery device is most appropriate? a.Nasal cannula b.Simple face mask c.Partial non-rebreather mask d.Non-rebreather mask
answer
ANS: A Nasal cannulas deliver oxygen from 1 to 6 L/min. A patient with COPD should never receive more than 3 L/min because this decreases the drive to breathe, resulting in hypoventilation. All other devices are intended for flow rates greater than 6 L/min.
question
The nurse is assessing a patient with a right pneumothorax. Which finding would the nurse expect? a.Bilateral expiratory crackles b.Absence of breath sounds on the right side c.Right-sided wheezes on inspiration d.Trachea deviated to the right
answer
ANS: B A right pneumothorax is a collapsed lung; therefore, no breath sounds should be heard on that side. Crackles indicate pneumonia. Wheezes are asthma related. A collapsed right lung would cause the trachea to deviate to the left.
question
The nurse is examining a patient who is admitted to the emergency department with severe elbow pain. Of the following situations, which would cause the nurse to suspect a ligament tear or joint fracture? a.Range of motion of the elbow is limited. b.Joint motion is greater than normal. c.The patient has arthritis. d.The elbow cannot be moved (frozen).
answer
ANS: B Increased mobility (beyond normal) of a joint may indicate connective tissue disorders, ligament tears, or possible joint fractures. Limited range of motion often indicates inflammation such as arthritis, fluid in the joint, altered nerve supply, or contractures (frozen joints).
question
The patient is admitted with a stroke. The outcome of this disorder is uncertain, but the patient is unable to move his right arm and leg. The nurse understands that a.Active range of motion is the only thing that will prevent contractures from forming. b.Passive range of motion must be instituted to help prevent contracture formation. c.Range-of-motion exercises should be started 2 days after the patient is stable. d.Range-of-motion exercises should be done on major joints only.
answer
ANS: B When patients cannot participate in active range of motion, the nurse must institute passive range of motion to maintain joint mobility and prevent contractures. Passive range of motion can be substituted for active when needed. For the patient who does not have voluntary motor control, passive range-of-motion exercises are the exercises of choice. Unless contraindicated, the nursing care plan includes exercising each joint (not just major joints) through as nearly a full range of motion as possible. Initiate passive range-of-motion exercises as soon as the patient loses the ability to move the extremity or joint.
question
The nurse is developing a plan of care for a patient diagnosed with activity intolerance. Of the following strategies, which has the best chance of maintaining patient compliance? a.Performing 20 minutes of aerobic exercise daily with 10 minute warm-up and cool-down periods b.Instructing the patient to use an exercise log to record day, time, duration, and responses to exercise activity c.Instructing the patient on the evils of not exercising, and getting her to take responsibility for her current health status d.Arranging for the patient to join a gym that she will have to pay, for so that she does not need to depend on insurance
answer
ANS: B Keeping a log may increase adherence to an exercise prescription. Cross-training (combination of exercise activities) provides variety to combat boredom and increases the potential for total body conditioning as opposed to daily aerobic exercise. "Blaming" a patient for his or her health status is usually counterproductive. Instead, the nurse should instruct the patient about the physiological benefits of a regular exercise program. Developing a plan of exercise that the patient may perform at home may improve compliance.
question
The nurse is caring for a patient who has a temperature reading of 100.4° F (38° C). His last two temperature readings were 98.6° F (37° C) and 96.8° F (36° C). The nurse should a.Call the physician and anticipate an order to treat the fever. b.Assume that the patient has an infection and order blood cultures. c.Wait an hour and recheck the patient's temperature. d.Be aware that temperatures this high are harmful and affect patient safety.
answer
ANS: C Waiting an hour and rechecking the patient's temperature would be the most appropriate action in this case. A fever usually is not harmful if it stays below 102.2° F (39° C), and a single temperature reading does not always indicate a fever. In addition to physical signs and symptoms of infection, a fever determination is based on several temperature readings at different times of the day compared with the usual value for that person at that time. Mild temperature elevations enhance the body's immune system by stimulating white blood cell production. Usually, staff nurses do not order blood cultures, and nurses should base actions on knowledge, not on assumptions.
question
The nurse is caring for a patient who has an elevated temperature. The nurse understands that a.Fever and hyperthermia are the same thing. b.Hyperthermia occurs when the body cannot reduce heat loss. c.Hyperthermia is an upward shift in the set point. d.Hyperthermia occurs when the body cannot reduce heat production.
answer
ANS: D Fever and hyperthermia are not the same things. An elevated body temperature related to the body's inability to promote heat loss or reduce heat production is hyperthermia. Fever is an upward shift in the set point. Hyperthermia is not a shift in the set point.
question
The patient is admitted with shortness of breath and chest discomfort. Which of the following laboratory values could account for the patient's symptoms? a.Hemoglobin level of 8.0 b.Hematocrit level of 45% c.Red blood cell count of 5.0 million/mm3 d.Pulse oximetry of 90%
answer
ANS: A The concentration of hemoglobin reflects the patient's capacity to carry oxygen. Normal hemoglobin levels range from 10 to 18 g/100 mL in males and from 12 to 16 g/100 mL in females. Hemoglobin of 8.0 is low and indicates a decreased ability to deliver oxygen to meet bodily needs. All other values in the selection are considered normal.
question
Of the following values, which value would be considered prehypertension? a.98/50 in a 7-year-old child b.115/70 in an infant c.140/90 in an older adult d.120/80 in a middle-aged adult
answer
ANS: D An adult's blood pressure tends to rise with advancing age. The optimal blood pressure for a healthy, middle-aged adult is less than 120/80. Values of 120 to 139/80 to 89 mm Hg are considered prehypertension. Blood pressure greater than 140/90 is defined as hypertension. Blood pressure of 98/50 is normal for a child, whereas 115/70 can be normal for an infant.
question
When taking the pulse of an infant, the nurse notices that the rate is 145 beats/min and the rhythm is regular. The nurse realizes that his rate is a.Normal for an infant. b.The proper rate for a toddler. c.Too slow for an infant. d.The same as that of a normal adult.
answer
ANS: A The normal rate for an infant is 120 to 160 beats/min. The rate obtained (145 beats/min) is within the normal range for an infant. The normal rate for a toddler is between 90 and 140 beats/min. The normal rate for an adult is between 60 and 100 beats/min.
question
The nurse is caring for a newborn infant in the hospital nursery. She notices that the infant is breathing rapidly but is pink, warm, and dry. The nurse knows that the normal respiratory rate for a newborn is _____ breaths per minute. a.30 to 60 b.25 to 32 c.16 to 19 d.12 to 20
answer
ANS: A The acceptable respiratory rate range for a newborn is 30 to 60 breaths per minute. An infant (6 months) is expected to have a rate between 30 and 50 breaths per minute. A toddler's respiratory range is 25 to 32 breaths per minute. A child should breathe 20 to 30 times a minute. An adolescent should breathe 16 to 19 times a minute. An adult should breathe 12 to 20 times a minute.
question
While attempting to obtain oxygen saturation readings on a toddler, what should the nurse do? a.Place the sensor on the earlobe. b.Place the sensor on the bridge of the nose. c.Determine whether the toddler has a tape allergy. d.Ignore any variation between the oximeter pulse rate and the patient's apical pulse rate.
answer
ANS: C The nurse should determine whether the patient has latex allergy because disposable adhesive probes should not be used on patients with latex allergies. Earlobe and bridge of the nose sensors should not be used on infants and toddlers because of skin fragility. Oximeter pulse rate and the patient's apical pulse rate should be the same. Any difference requires re-evaluation of oximeter sensor probe placement and reassessment of pulse rates.
question
The nurse is preparing to assess the blood pressure of a 3-year-old. How should the nurse proceed? a.Choose the cuff that says "Child" instead of "Infant." b.Obtain the reading before the child has a chance to "settle down." c.Use the diaphragm portion of the stethoscope to detect Korotkoff sounds. d.Explain to the child what the procedure will be.
answer
ANS: D Preparing the child for the blood pressure cuff's unusual sensation increases cooperation. Most children will understand the analogy of a "tight hug on your arm." Different arm sizes require careful and appropriate cuff size selection. Do not choose a cuff based on the name of the cuff. An "Infant" cuff is too small for some infants. Readings are difficult to obtain in restless or anxious infants and children. Allow at least 15 minutes for children to recover from recent activities and become less apprehensive. Korotkoff sounds are difficult to hear in children because of low frequency and amplitude. A pediatric stethoscope bell is often helpful.
question
The nurse is caring for a patient who complains of feeling light-headed and "woozy." The nurse checks the patient's pulse and finds that it is irregular. The patient's blood pressure is 100/72. It was 113/80 an hour earlier. What should the nurse do? a.Call the physician immediately. b.Perform an apical/radial pulse assessment. c.Apply more pressure to the radial artery to assess the pulse. d.Use his thumb to detect the patient's pulse.
answer
ANS: B If the pulse is irregular, do an apical/radial pulse assessment to detect a pulse deficit. If pulse count differs by more than 2, a pulse deficit exists, which sometimes indicates alterations in cardiac output. The nurse needs to gather as much information as possible before calling the physician. The radial pulse is more accurately assessed with moderate pressure. Too much pressure occludes the pulse and impairs blood flow. Fingertips are the most sensitive parts of the hand to palpate arterial pulsations. The thumb has a pulsation of its own that interferes with accuracy.
question
Of the following sites, which are used for obtaining a core temperature? (Select all that apply.) a.Oral b.Rectal c.Tympanic d.Axillary e.Pulmonary artery
answer
ANS: C, E Intensive care units use the core temperatures of the pulmonary artery, esophagus, and urinary bladder. Because the tympanic membrane shares the same arterial blood supply as the hypothalamus, the tympanic temperature is a core temperature. Oral, rectal, axillary, and skin temperature sites rely on effective blood circulation at the measurement site.
Get an explanation on any task
Get unstuck with the help of our AI assistant in seconds
New